Becker: EXAM #1, REG 1, MODULES 1-6

Réussis tes devoirs et examens dès maintenant avec Quizwiz!

Mark and Molly met at a New Year's Eve party held December 31, Year 1. They instantly bonded, fell madly in love, and were married at 11:38 p.m that night. Identify Mark's filing status for Year 1. (1) Single (2) Married filing jointly (3) Head of household (4) Surviving spouse

Choice "2" is correct. Mark and Molly were married as of midnight on December 31, Year 1. Therefore, Mark's only options are to file as married either jointly or separately, and because "jointly" is the only option presented that qualifies, it is the correct choice.

Jay received a court award for damages to his personal reputation by the National Gossip. He also received punitive damages. Which of the following statements is true? (1) None of the damages are taxable (2) All of the damages are taxable (3) Only the compensatory damages are taxable (4) The compensatory damages are not taxable, but the punitive damages are taxable

Choice "2" is correct. Personal reputation awards and punitive damage awards are both included in taxable income.

Adams owns a second residence that is used for both personal and rental purposes. During the current year, Adams used the second residence for 50 days and rented the residence for 200 days. Which of the following statements is correct? (1) Depreciation may not be deducted on the property under any circumstances (2) A rental loss may be deducted if rental-related expenses exceed rental income (3) Utilities and maintenance on the property must be divided between personal and rental use (4) All mortgage interest and taxes on the property will be deducted to determine the property's net income or loss

Choice "3" is correct. Because the second property was personally used more than 14 days, any net loss from the rental of the property will be disallowed. All related expenses must be prorated between the personal use portion and the rental activity portion. Prorated depreciation is permitted for the rental activity.

Which of the following is an itemized deduction? (1) Educator expenses (2) Moving expenses for a move due to change in employment (3) Qualified charitable contributions (4) Roof repair due to regular wear and tear

Choice "3" is correct. Qualified charitable contributions are an itemized deduction subject to AGI limitations. Choice "1" is incorrect. Educator expenses are an adjustment to arrive at AGI. Choice "2" is incorrect. Moving expenses are only deductible if pursuant to military order. Choice "4" is incorrect. Regular maintenance is not an itemized deduction. In addition, if the repair was due to a casualty loss, it would only be deductible as an itemized deduction if attributable to a federal disaster.

Which one of the following statements is correct with regard to an individual taxpayer who has elected to amortize the premium on a bond that yields taxable interest? (1) The amortization is treated as an itemized deduction (2) The amortization is not treated as a reduction of taxable income (3) The bond's basis is reduced by the amortization (3)The bond's basis is increased by the amortization

Choice "3" is correct. The bond's basis is reduced by the amortization of the premium. Choice "1" is incorrect. The amortization of the premium is an offset to interest income on the bond rather than a separate interest deduction. Choice "2" is incorrect. The amortization of the premium will reduce taxable income. Choice "4" is incorrect. The bond's basis will be decreased by the amortization.

Dylan died on March 2, Year 1. Ann his wife, and Lena, their daughter survive. Ann filed a joint return in Year 1. Lena, age 19 in Year 4, is a college student and continues to live at home with her mother. She works part-time, earning wages of less than the personal exemption amount for the year. What is Ann's filing statues for Year 4? (1) Single (2) Surviving spouse (3) Married, filing jointly (4) Head of household

Choice "4" is correct. Her dependent child, Lena, is living with her. Choice "1" is incorrect. While it is true that Ann is single at December 31, Year 4, the head of household status is available to her and is more advantageous. Choice "2" is incorrect. The surviving spouse is only available for the two years after death, provided a dependent child is living with you. The Year 4 is three years after death. Choice "3" is incorrect. At December 31, Year 4, Ann is not married, and hadn't been married at all in Year 4; therefore, she cannot file as married.

Marty Smarty is a CPA in private practice. For the current year ended December 31, Marty had the following items of income and expenses with respect to his CPA practice: Gross Revenue $275,000 Rent Expense $24,000 Wages paid to employees $60,000 Wages paid to Marty $120,000 Payroll taxes for employees $5,000 Payroll taxes for Marty (estimates) $9,000 Supplies Expense $10,000 Insurance Expense $8,000 Depreciation Expense $15,000 Business Meals $4,000 Health Insurance for employees $5,000 Health insurance for Marty $2,000 Business bad debt loss (allowance) $3,000 State income taxes for the business $10,000 What is Marty's taxable income on Schedule C for the CPA practice for the current year? (1) $120,000 (2) $130,000 (3) $144,000 (4) $146,000

Choice "4" is correct. The answer is calculated as follows: Gross revenue 275,000 Rent expense (24,000) Wages paid to employees (60,000) Wages paid to Marty [considered a draw, not an expense] Payroll taxes for employees (5,000) Payroll taxes for Marty (estimates) [not deducted on Sch. C, 50% is an adjustment] Supplies expense (10,000) Insurance expense (8,000) Depreciation expense (15,000) Business meals (2,000) [only 50% deductible] Health insurance for employees (5,000) Health insurance for Marty [not deducted on Sch. C, 100% is an adjustment] Business bad debt loss (allowance) [direct write-off only for accrual basis taxpayers] State income taxes for the business [not deducted on Sch. C, an itemized deduction] Schedule C 146,000

Robbe, a cash basis single taxpayer, reported $50,000 of adjusted gross income last year and claimed itemized deductions of $5,500, consisting solely of $5,500 of state income taxes paid last year. Robbe's itemized deduction amount exceeded the standard deduction available to single taxpayer for last year by $1,150. In the current year, Robbe received a $1,500 state tax refund relating to the prior year. What is the proper treatment of the state tax refund? (1) Include none of the refund in come in the current year (2) Include $1,150 in income in the current year (3) Include $1,500 in income in the current year (4) Amend the prior-year's return and reduce the claimed itemized deductions for that year

Rule: IRC Section 111 provides that gross income does not include income attributable to the recovery during the taxable year of any amount deducted in any prior taxable year to the extent such amount did not reduce the amount of tax previously imposed (the tax benefit rule). Choice "2" is correct. Under the tax benefit rule, an itemized deduction recovered in a subsequent year is included in income in the year recovered. In this question, only $1,150 of the state income taxes was actually deducted as an itemized deduction last year. The recovery is thus limited in the amount actually deducted (and not to the entire amount of the state tax refund). Choice "1" is incorrect. The amount deducted, not $0, is included in income in the current year. Choice "3" is incorrect. The amount originally deducted, not necessarily the entire amount of the refund, is included in income in the current year. Choice "4" is incorrect. The amount deducted is included in income in the current year. It is not necessary to amend the prior year's return.

Which of the following is considered a specified service trade or business (SSTB) for purposes of the qualifying business income deduction? (1) Accounting firm (2) Manufacturing company (3) Engineering firm (4) Architectural services

Choice "1" is correct. Accounting services are considered an SSTB for purposes of the qualified business income deduction. Choice "2" is incorrect. A manufacturing firm is a qualified trade or business (QTB) and not an SSTB. Choice "3" is incorrect. An engineering firm is specifically excluded from the definition of an SSTB. Choice "4" is incorrect. Architectural services are specifically excluded from the definition of an SSTB.

A 33-year-old taxpayer withdrew $30,000 (pretax) from a traditional IRA. The taxpayer has a 33% effective tax rate and a 35% marginal tax rate. What is the total tax liability associated with the withdrawal? (1) $10,000 (2) $10,500 (3) $13,000 (4) $ 13,500

Rule: Generally, unless an exception applies, retirement money cannot be withdrawn until the individual reaches the age of 59 ½. If retirement money (without an exception) is withdrawn before the age of 59 ½, the premature distribution is subject to a 10% penalty tax (in addition to the applicable regular income tax that applies to all distributions of traditional IRA money). Choice "4" is correct. The taxpayer is under the age of 59 ½, and the facts do not indicate that an exception applies; therefore, the taxpayer is subject to the 10% penalty on the IRA distribution in addition to the regular income tax. The regular income tax that applies is the marginal rate (the rate for the next dollar of taxable income). The effective tax rate is simply the total tax divided by the total taxable income. In this case, the taxpayer would have to pay the regular tax on the distribution at the 35% marginal rate PLUS the 10% penalty on early distribution without an exception. The calculation to arrive at the total tax associated with the withdrawal follows: Regular Income Tax 30,000 × 35% 10,500 Penalty Tax 30,000 × 10% 3,000 Total Tax 13,500 Choice "1" is incorrect. This answer option assumes the effective income tax rate (rounded, assuming 33.33%) applied to the $30,000 distribution. It uses the incorrect tax rate (the marginal rate should be used) and omits the inclusion of the applicable 10% penalty tax. [$30,000 × 33.33% = $10,000] Choice "2" is incorrect. This answer option includes the $30,000 distribution multiplied by the (proper) marginal tax rate, but it omits the inclusion of the applicable 10% penalty tax. [$30,000 × 35% = $10,500] Choice "3" is incorrect. This answer option assumes the effective income tax rate (rounded, assuming 33.33%) applied to the $30,000 distribution plus the applicable 10% penalty tax [($30,000 × 33.33%) + ($30,000 × 10%) = $13,000]. It uses the incorrect tax rate (the marginal rate should be used).

Smith has an adjusted gross income (AGI) of $120,000 without taking into consideration $40,000 of losses from rental real estate activities. Smith actively participates in the rental real estate activities. What amount of the rental losses may Smith deduct in determining taxable income? (1) $0 (2) $15,000 (3) $20,000 (4) $40,000

Choice "2" is correct. Generally, none of the passive losses from real estate are deductible against nonpassive income. However, Smith actively participates, which means that the "mom and pop" exception of up to $25,000 will apply. This exception is phased out over AGI of $100,000 through $150,000. That is 50 cents on the dollar. Smith's AGI is $120,000. That is $20,000 into the phaseout range. So $10,000 of the $25,000 is phased out and Smith may deduct $15,000 of the $40,000 passive loss.

Aston and Becker are equal partners in AB Partnership. In the tax year, the ordinary income of the partnership is $20,000, and the partnership has a long-term capital gain of $12,000. Aston's basis inAB was $40,000, and he received distributions of $5,000 during the year. What is Aston's share of AB's ordinary income? (1) $10,000 (2) $15,000 (3) $16,000 (4) $18,500

Choice "1" is correct. Aston's share of ordinary income is $10,000 (50 percent partnership percentage × $20,000 partnership ordinary income). Choice "2" is incorrect. Ordinary income does not include distributions. Choice "3" is incorrect. Ordinary income does not include long-term capital gains. Choice "4" is incorrect. Ordinary income does not include distributions or long-term capital gains.

The following Year 1 annual report was received by Clark from the qualified defined contribution plan provided by Clark's employer: Beginning balance $12,700 Employer contribution 600 Plan earnings 250 Ending balance $13,550 What income must be included in Clark's gross income for Year 1? (1) $0 (2) $250 (3) $600 (4) $850

Choice "1" is correct. Employer contributions to a qualified traditional defined contribution retirement plan and earnings on the amounts contributed are not taxable income to the employee until distributed. Choice "2" is incorrect. The plan earnings in Year 1 are not taxable income to the employee until distributed. Choice "3" is incorrect. The employer contribution of $600 in Year 1 is not taxable income to the employee until distributed. Choice "4" is incorrect. The $850 employer contribution and plan earnings in Year 1 are not taxable income to the employee until distributed.

Which of the following statements regarding passive activity losses is true? (1) A net passive activity loss may be deducted against wages (2) Losses on rental property are always considered passive (3) A passive activity is one in which the taxpayer does not materially participate (4) Expenses related to passive activities may be deducted from passive activity income and portfolio income

Choice "3" is correct. A passive activity is an activity in which the taxpayer does not materially participate. Choice "1" is incorrect. Passive activity expenses are only deductible from passive activity income. Choice "2" is incorrect. Although rental activities are usually considered passive, they are not passive if the taxpayer is a real estate professional. Choice "4" is incorrect. Passive activity expenses are only deductible from passive activity income.

Which of the following is true about the qualifying business income (QBI) deduction for taxpayers with taxable income above the taxable income limitations? (1) If the taxpayer is a specified service trade or business (SSTB), no QBI deduction is allowed (2) If the taxpayer is a qualified trade or business (QTB), W-2 wage and property limitations do not apply (3) If the taxpayer is a qualified trade or business (QTB), W-2 wage and property limitations are phased in (4) If the taxpayer is a specified service trade or business (SSTB), W-2 wage and property limitations apply

Choice "1" is correct. For a specified service trade or business (SSTB) over the taxable income limitation, no QBI deduction is allowed. Choice "2" is incorrect. For a qualified trade or business (QTB) over the taxable income limitation, W-2 wage and property limitations do apply. Choice "3" is incorrect. For a qualified trade or business (QTB) over the taxable income limitation, W-2 wage and property limitations apply in full and are not phased in. Choice "4" is incorrect. For a specified service trade or business (SSTB) over the taxable income limitation, no QBI deduction is allowed.

A guaranteed payment by a partnership to a partner for services rendered, may include an agreement to pay: I. A salary of $10,000 monthly without regard to partnership income. II. A 30% interest in partnership profits. (1) I only (2) II only (3) Both I and II (4) Neither I nor II

Choice "1" is correct. I. A guaranteed payment is a salary or other payment to a partner that is not calculated with respect to partnership income. II. Since the 25% interest is calculated with respect to partnership profits, it is not a guaranteed payment.

The percentage-of-completion method is required for tax purposes for long-term contracts unless an exemption exists for the taxpayer. Which of the following situations does not describe an exemption from using the percentage-of-completion method for a long-term contract? (1) A long-term construction contract that includes land with 20% of the contract cost for construction of property on the land (2) Services performed under a maintenance agreement (3) A project completed in an eighteen-month period by a contractor with average annual gross receipts of $8 million for the last five years (4) Services of an engineer contracted by a builder responsible for a two-year project

Choice "1" is correct. A long-term construction contract that includes land and less than 10% of the total contract cost relates to the actual construction of property on the land is exempt from the percentage-of-completion method reporting requirement. Because the contract has 20% of the cost related to construction of property, the contract is not exempt. Choice "2" is incorrect. Services performed under a maintenance agreement are exempt from percentage-of-completion reporting requirements. Choice "3" is incorrect. Projects that are expected to last no more than two years and are performed by a taxpayer with average annual gross receipts not exceeding $25 million for the three previous years are exempt from percentage-of-completion reporting requirements. Choice "4" is incorrect. The service of an engineer contracted by a builder on a long-term project is exempt from the percentage-of-completion reporting requirement.

The term active participation for a passive activity loss is relevant in relation to: (1) Rental real estate activities (2) Working interests in oil and gas properties (3) Passive activities in which the taxpayer materially participates (4) Passive activities in which the taxpayer does not materially participate

Choice "1" is correct. Active participation in rental real estate activities allows the taxpayer to deduct losses from the rental activities against other income, subject to limitation. Choice "2" is incorrect. Active participation is not relevant to working interests in oil and gas properties. Choice "3" is incorrect. Material participation requires a higher level of taxpayer involvement than required by active participation. Choice "4" is incorrect. Active participation is not relevant to all passive activities.

The Clarks have a 21 -year-old son, Alex, who is a full-time student at the state university. Alex received $10,000 in scholarships this year for academic achievement. He also works part time at the university bookstore and earned $5,400 this year. The Clarks paid $7,000 to support Alex this year. Alex was home for two months in the summer and at school for the rest of the year. Alex used the scholarship, the earnings from the part-time job, and the money from his parents as his only source of support this year. Which of the following definitions does Alex meet for the Clarks? (1) Qualifying child (2) Qualifying relative (3) Exemption (4) Qualifying person

Choice "1" is correct. Alex meets the definition of qualifying child for the Clarks. He meets the close relative test because he is their son. He is under the age of 24 and is a full-time student, so he meets the age limit. He meets the residency requirements because his principal place of abode is his parents' home since he was only away as a student. He also does not provide more than half of his own support. The scholarship does not count as support provided by Alex. Even though Alex's grandparents provide $7,000 of support, Alex is a qualifying child of his parents because Alex does not provide more than half of his own support. Choice "2" is incorrect. Alex meets the definition of qualifying child of the Clarks. He does not meet the definition of qualifying relative because his gross income is too high $4,300 for 2020. Choice "3" is incorrect. Alex is not an exemption for the Clarks. Exemptions for dependents are not allowed for tax years 2018 and later. Choice "4" is incorrect. Qualifying person is not a dependency definition type.

In a 2017 divorce settlement, the ex-husband was required by court order to pay his ex-wife $36,000 in alimony. She received $25,000 in cash, a painting valued at $10,000, and the use of his beach house, valued at $3,000. What amount of gross income should she report as alimony? (1) $25,000 (2) $35,000 (3) $36,000 (4) $38,000

Choice "1" is correct. Alimony includes only payments received in cash or its equivalent (e.g., the payment of bills on behalf of the ex-spouse). Alimony received pursuant to a divorce executed on or before December 31, 2018, is included in gross income of the recipient. Choice "2" is incorrect. The painting would not be included in alimony because it is not a payment in cash or its equivalent. Choice "3" is incorrect. The amount of alimony ordered by the court does not determine the amount of alimony that is considered gross income for tax purposes. Choice "4" is incorrect. Neither the painting nor the use of the beach house would count as alimony because they are not payments in cash or its equivalent.

Bartlet owns a manufacturing business and participates in the business. Which of the following conditions would cause the business to be considered a nonpassive activity for Bartlet? (1) Bartlet participates in the business for more than 500 hours during a year (2) The business made a profit in any three of the last five years that preceded the current year (3) The business has at least 10 employees who, individually or collectively, work for the business more than 1,000 hours in a year (4) Bartlet files an election with the IRS postponing nonpassive activity classification

Choice "1" is correct. Bartlet's participation in the business would be considered nonpassive if he is considered to materially participate in the business. One test used for determining material participation in a business is if the individual participates in the activity for more than 500 hours during the year. Choice "2" is incorrect. Whether an activity made a profit in any three of the last five years preceding the current year is a test for determining if the activity is a business as compared to a hobby. Choice "3" is incorrect. This is not a test that would be used to determine if the activity is a nonpassive activity for Bartlet. Choice "4" is incorrect. This is not a test to determine whether an individual materially participates in a business.

On March 1 of year 0, Judy was granted an incentive stock option (ISO) to purchase 50 shares of her employer's stock for $10 per share. The FMV of the stock on the date of the grant was $12 per share. On May 1 of year 1, Judy exercised her option when the stock was selling for $15 per share. On July 1 of year 2, Judy sold all of the shares for $20 per share. What amount and character of income does Judy recognize in year 0? (1) $0 (2) $500, ordinary (3) $500, capital (4) $600, ordinary

Choice "1" is correct. Because the stock option qualified as an incentive stock option, Judy does not recognize any income in the year the options were granted (Year 0). Choices "2", "3", and "4" are incorrect. The grant of qualified incentive stock options is not a taxable event.

Cobb, an unmarried individual, had an adjusted gross income of $200,000 in 2017 before any IRA deduction, taxable Social Security benefits, or passive activity losses. Cobb incurred a loss of $30,000 in 2017 from rental real estate in which he actively participated. What amount of loss attributable to this rental real estate can be used in 2017 as an offset against income from nonpassive sources? (1) $0 (2) $12,500 (3) $25,000 (4) $30,000

Choice "1" is correct. Cobb may not use any of the loss attributable to his rental real estate as an offset against income from nonpassive sources in the current year because he does not qualify for the "Mom and Pop" exception. Under this exception, up to $25,000 of passive losses and the deduction equivalent of tax credits that are attributable to rental real estate may be used as an offset against income from nonpassive sources. This $25,000 allowance is reduced, but not below zero, by 50% of the amount by which the individual's modified AGI exceeds $100,000. The $25,000 is therefore completely phased out when modified AGI reaches $150,000. Because Cobb's AGI was $200,000, he did not qualify for the exception. Choices "2", "3", and "4" are incorrect. Rental activities are passive activities and generally are not allowed to use any of the loss attributable to the rental activity to offset any income produced from nonpassive sources. There is a limited exception in the case of losses from rental real estate in which the taxpayer actively participates, but Cobb did not qualify for it.

Which of the following costs are subject to the Uniform Capitalization Rules of Code Sec. 263A for manufactured tangible personal property? (1) Off-site storage (2) Advertising (3) Research (4) Marketing

Choice "1" is correct. Costs required to be capitalized under the uniform capitalization rules include direct materials, direct labor, and applicable indirect costs. Applicable indirect costs include utilities, warehousing costs, repairs, maintenance, indirect labor, rents, storage, depreciation and amortization, insurance, pension contributions, engineering and design, repackaging, spoilage and scrap, and administrative supplies. Choice "2" is incorrect. Costs not required to be capitalized include selling, advertising, and marketing expenses, certain general and administrative expenses, research, and officer compensation not attributed to production services. Choice "3" is incorrect. Costs not required to be capitalized include selling, advertising, and marketing expenses, certain general and administrative expenses, research, and officer compensation not attributed to production services. Choice "4" is incorrect. Costs not required to be capitalized include selling, advertising, and marketing expenses, certain general and administrative expenses, research, and officer compensation not attributed to production services.

Rich is a cash basis self-employed air conditioning repairman with current year gross business receipts of $20,000. Rich's cash disbursements were as follows: Air-conditioning parts $2,500 Yellow pages listing 2,000 Estimated federal income taxes on self-employment income 1,000 Business long-distance telephone calls 400 Charitable contributions 200 What amount should Rich report as net self-employment income? (1) $15,100 (2) $14,900 (3) $14,100 (4) $13,900

Choice "1" is correct. Deductions to arrive at net self-employed income include all necessary and ordinary expenses connected with the business. Estimated federal income tax payments are not an expense. Charitable contributions by an individual are only deductible as an itemized deduction on Schedule A. This assumes the contribution was not made with the "expectation of commensurate financial return." Receipts 20,000 Parts (2,500) Listing (2,000) Telephone (400) Net self-employment income 15,100 Choice "2" is incorrect. Charitable contributions are an itemized deduction unless there is an expectation of commensurate financial return. Choice "3" is incorrect. Federal income taxes paid are not a deductible expense. Choice "4" is incorrect. Charitable contributions are an itemized deduction unless there is an expectation of commensurate financial return. Federal income taxes paid are not a deductible expense.

Under the uniform capitalization rules applicable to property acquired for resale, which of the following costs should be capitalized with respect to inventory if no exceptions are met? Repackaging costs; Off-site storage costs (1) Yes; Yes (2) Yes; No (3) No; Yes (4) No; No

Choice "1" is correct. Direct material, direct labor, and factory overhead (applicable indirect costs) are capitalized with respect to inventory under the uniform capitalization rules for property acquired for resale. Applicable indirect costs include depreciation and amortization, insurance, supervisory wages, utilities, spoilage and scrap, design expenses, repair and maintenance and rental of equipment and facilities (including offsite storage), some administrative costs, costs of bonus and other incentive plans, and indirect supplies and other materials (including repackaging costs).

In which of the following scenarios would the head of household filing status be available to the taxpayer? (1) A single taxpayer maintains a separate home for his parent, who qualifies as a dependent (2) A taxpayer with no dependents is the surviving spouse of an individual who died in the current year (3) An unmarried taxpayer maintains a household with a 28-year-old son, who earned $10,000 during the tax year (4) A single taxpayer maintains a household that is the principal home for five months of the year for his disable child

Choice "1" is correct. Head of household filing status is available to a single taxpayer who maintains a separate home for a dependent parent. To qualify for head of household filing status, a taxpayer must be unmarried as of the last day of the tax year and maintain a home that is the principal residence of a qualifying person for more than half of the tax year. A qualifying person includes a dependent child, parent, or relative. A dependent parent is not required to live with the taxpayer, provided the taxpayer maintains a home that was the principal residence of the parent for the entire year. Choice "2" is incorrect. A taxpayer who has no dependents is not entitled to head of household filing status. Because the taxpayer's spouse died in the current year, the taxpayer is entitled to married filing jointly status for the current year. Choice "3" is incorrect. The taxpayer is not entitled to head of household filing status. Her filing status is single. The taxpayer is unmarried and maintains a home for her son, but her son is not a dependent child or dependent relative. Her son is not a qualifying child dependent because he is over the age limit (under age 19, or under age 24 in the case of a full-time student). He is not a qualifying relative dependent because his gross income of $10,000 is more than the gross income limit of $4,300 (2020). Choice "4" is incorrect. The taxpayer is not entitled to head of household filing status. His filing status is single. To qualify for head of household status, the taxpayer must maintain a home that is the principal residence of a qualifying person for more than half the year. Because the taxpayer maintains a household that is the principal home for his disabled child for only five months of the year, his son is not a dependent child or relative and he does not qualify for head of household status.

Donner Corporation, an S corporation, had an ordinary loss of $73,000 for the current year ended December 31. At January 1 of the current year, Cracraft, an individual, owned 50% of Donner's stock. Cracraft held the stock for 90 days during the year before selling the entire 50% interest to an unrelated third party. Cracraft's properly-calculated basis for the stock on the date of sale was $20,000, and Cracraft sold the stock for $10,000. Cracraft was a full-time employee of Donner until the stock was sold. Cracraft's share of Donner's current year loss was: (1) $9,000 (2) $10,000 (3) $18,000 (4) $36,500

Choice "1" is correct. If ownership interests in an S corporation change within the taxable year, the income and/or loss to be allocated among the various shareholders will be made on a "per share/per day" basis. S corporation loss for the year (73,000) Divided by 365 days in a year ÷ 365 Losses per day during the year (200) Number of days Cracraft was a shareholder × 90 Loss allocated to 90 days (18,000) Cracraft's ownership interest × 50% Loss allocated to Cracraft for the year ( 9,000) Choice "2" is incorrect. This amount represents Cracraft's capital loss on the sale of Donner's stock [$10,000 selling price − $20,000 basis]. The allocated losses of $9,000 would already have been included in the basis calculation to arrive at the $20,000. Choice "3" is incorrect. This amount represents 100% of Donner's losses for the 90 days Cracraft held the stock. Cracraft only owned 50% of the stock for those 90 days. Choice "4" is incorrect. This amount represents 50% of Donner's total loss for the year. Cracraft only held the stock for 90 days during the year, and the losses must be allocated based on the number of days he held the stock during the year (as well as the percentage ownership).

Briana has various items of income as follows: W-2 wages $24,000 Interest and dividends $3,000 Sole proprietorship income on Schedule C $98,000 Income from an S corporation from Schedule K-1 $12,000 Income as a limited partner from a limited partnership from Schedule K-1 $10,000 For purposes of self-employment tax, what are the net earnings from self-employment? (Note: Please answer before the required 92.35% calculation on Schedule SE.) (1) $98,000 (2) $22,000 (3) $27,000 (4) $10,000

Choice "1" is correct. Income subject to self-employment includes amounts from an unincorporated sole proprietorship (Schedule C) and general partnerships. It does not include W-2 wages, interest, dividends, income from an S corporation, or income as a limited partner from a limited partnership. The only amount here that qualifies is the $98,000 sole proprietorship income.

Harry has various items of income as follows: W-2 wages $24,000 Interest and dividends $3,000 Rental real estate on Schedule E $8,000 Income from an S corporation from Schedule K-1 $12,000 Income from a general partnership from Schedule K-1 $10,000 For purposes of the self-employment tax, what are the net earnings from self-employment? (Note: Please answer before the required 92.35% calculation performed on Schedule SE.) (1) $10,,000 (2) $22,000 (3) $30,000 (4) $57,000

Choice "1" is correct. Income subject to self-employment includes amounts from an unincorporated sole proprietorship (Schedule C) and general partnerships. It does not include W-2 wages, interest, dividends, rental real estate income, or income from an S corporation. The only amount here that qualifies is the $10,000 from the general partnership.

Gail and Mark James contributed to the support of their two children, Jack and Jill, as well as Mark's mother, Betty. Jack is a 19-year-old full time student who earned $5,000 this year working at a coffee shop on campus. Jill is 24 years old and worked full-time as a librarian and earned $25,000. Jack comes home during the summer and holidays. Jill lives at home year-round. Betty lives in an apartment in town and received $2,000 in municipal bond interest, $6,000 dividend income, and $4,000 in nontaxable Social Security benefits. Jack, Jill, and Betty are U.S. citizens and unmarried. Gail and Mark provided more than half of the support for Jack, Jill, and Betty. How many people qualify as dependents on Gail and Mark's tax return? (1) One (2) Two (3) Three (4) Zero

Choice "1" is correct. Jack meets the CARES test for a qualifying child. Jill does not meet the CARES test for a qualifying child because she is 24 and not a full-time student. She fails the "1" age limit test of CARES. Jill also does not meet the SUPORT test because she earns more taxable income than the gross income threshold amount ("U" for under that amount). Betty does not meet the CARES test because she fails the "3" close relative and "1" age limit test. (The CARES test is for a qualifying child, not a qualifying relative.) Betty also fails the SUPORT test because her taxable income ($6,000) is not under the gross income threshold amount. Therefore, Gail and Mark James can claim one person as a dependent—Jack.

Paul paid the real estate taxes on his rental apartment building. The real estate taxes are: (1) A deduction to arrive at adjusted gross income (2) A deduction from adjusted gross income (3) A deduction from adjusted gross income, subject to a 10 percent AGI floor (4) Not deductible

Choice "1" is correct. Rental property expenses are reported on Schedule E (for AGI), and, subject to the passive activity loss provisions, the net number appears on Page 1 of the Form 1040 in arriving at AGI. Choice "2" is incorrect. Rental property expenses are not a deduction from adjusted gross income. They are taken into account in arriving at adjusted gross income. Choice "3" is incorrect. Rental property expenses are not an itemized deduction subject to a 10 percent AGI floor. Choice "4" is incorrect. See explanation for choice "1".

John earned $500,000 in his business during the current year, and his wife received investment income of $15,000. John provides more than half of the support of his 50-year-old widowed sister, who lives with John and earned $45,000 in salary. John also provides full support for his two children, an 18-year-old daughter and a 20-year-old son, who is a full-time college student. The family employs a live-in housekeeper and a live-in butler to assist them with their residence. Both the live-in housekeeper and the live-in butler provided all of their own support. How many people qualify as either a qualifying child or qualifying relative for John? (1) Two (2) Four (3) Five (4) Zero

Choice "1" is correct. John's two children qualify under the qualifying child rules (CARES). John's sister does not meet the age limit for QC and is not under the gross income limitation for QR. The butler and housekeeper both fail the support tests for both QC and QR because they provide all of their own support. Choice "2" is incorrect. John's children meet the definition of QC. The sister, housekeeper, and butler all fail both QC and QR. Choice "3" is incorrect. Only John's children meet dependency definitions. They both are qualifying children. The sister, housekeeper, and butler all fail both QC and QR. Choice "4" is incorrect. John's children meet QC rules.

This question is based on the following data: Sydney Corporation Income Statement For the year ended December 31, Year 5 Sales $1,800,000 Cost of sales (1,200,000) Gross margin 600,000 Operating expenses (500,000) Operating income 100,000 Other income: Gain on sale of investments $30,000 Life insurance policy proceeds 20,000 Dividends 6,000 Total 56,000 Other expenses: Contributions (18,000) Income before income tax $138,000 The life insurance policy proceeds represent a lump-sum payment in full as a result of the untimely death of Sydney's chief financial officer. Sydney was the owner and sole beneficiary of the policy; however, on receipt of the insurance proceeds, Sydney distributed the entire amount to the CFO's surviving spouse. On its Year 5 income tax return, Sydney should report taxable life insurance proceeds of: (1) $0 (2) $10,000 (3) $16,000 (4) $20,000

Choice "1" is correct. Life insurance proceeds on the life of an officer when the corporation is the owner and beneficiary are not reported as taxable income of the corporation. (Also note that any expense related to the premiums would not have been tax deductible for the corporation.)

As of December 31, the Mitchells were legally separated and maintained separate households for the entire year. The Mitchells have no children. What filing status should Mr. Mitchell claim for the year? (1) Single (2) Head of household (3) Married filing separately (4) Married filing jointly

Choice "1" is correct. Marital status for the tax year is determined as of the last day of the year. Taxpayers who are divorced or legally separated are considered unmarried. Mr. Mitchell is legally separated and does not have a qualifying dependent, so Mr. Mitchell's filing status for the year is single. Choice "2" is incorrect. The taxpayer does not qualify for head-of-household filing status because he does not maintain a home for a qualifying dependent. Choice "3" is incorrect. Because the taxpayer is legally separated, he is considered unmarried and does not qualify for married-filing-separately filing status. Choice "4" is incorrect. Because the taxpayer is legally separated, he is considered unmarried and does not qualify for married-filing-jointly filing status.

In Year 4 , after Mindy's three children have grown and moved out of the house, Mindy (unmarried) moved her mother, Mary, into an assisted living facility for which Mindy pays 75% of the cost. Mindy had not previously lived with Mary, and Mary paid for her own living expenses while she lived in her own home. What filing status should Mindy use for Year 4, assuming Mary moved into the assisted living facility on August 1, Year 4? (1) Single (2) Married filing jointly (3) Head of household (4) Surviving spouse

Choice "1" is correct. Mindy should file using the single status. She does not qualify for more favorable filing status. Choice "2" is incorrect. Mindy is not married. Choice "3" is incorrect. Mindy does not qualify for head of household status. Had Mary moved into the assisted living home for the entire year, Mindy would have been eligible for head of household status. Mindy did not provide more than half of Mary's support and for Year 4 is ineligible for head of household status. Choice "4" is incorrect. Mindy has not had a spouse die in the past two years.

Clark bought Series EE U.S. Savings Bonds after 1989. Redemption proceeds will be used for payment of college tuition for Clark's dependent child. One of the conditions that must be met for tax exemption of accumulated interest on these bonds is that the: (1) Purchaser of the bonds must be the sole owner of the bonds (or joint owner with his or her spouse) (2) Bonds must be bought be a parent (or both parents) and put in the name of the dependent child (3) Bonds must be bought by the owner of the bonds before the owner reaches the age of 24 (4) Bonds must be transferred to the college for redemption by the college rather than by the owner of the bonds

Choice "1" is correct. One of the conditions that must be met for tax exemption of accumulated interest on the bonds is that the purchaser of the bonds must be the sole owner of the bonds (or joint owner with his or her spouse). Other conditions include, for post-1989 bonds, the taxpayer is over age 24 when issued and is used to pay for higher education, reduced by tax-free scholarships, of the taxpayer, spouse, or dependents. Choice "2" is incorrect. The bonds must be bought and put in the name of the owner or co-owner, not in the name of the dependent child. Choice "3" is incorrect. The owner must be at least 24 years old before the bonds issue date. Choice "4" is incorrect. There is no requirement that the bonds must be transferred to the college for redemption by the college rather than by the owner of the bonds.

Passive activity losses of an individual taxpayer can generally be used to offset: (1) Income from the rental of a residence (2) A guaranteed payment received from a partnership (3) Dividend income from a foreign corporation (4) Interest income on U.S. Treasury notes

Choice "1" is correct. Passive activity losses (PALs) can only be offset against passive activity income, not active or portfolio income. Passive activities are trade or business activities in which the taxpayer does not materially participate. Rental real estate is a passive activity unless the taxpayer is a real estate professional. Choice "2" is incorrect. A guaranteed payment from a partnership is taxable active income. PALs can only offset passive activity income. Choice "3" is incorrect. Dividend income from a foreign corporation is taxable portfolio income. PALs can only offset passive activity income. Choice "4" is incorrect. Interest income from U.S. Treasury notes is taxable portfolio income. PALs can only offset passive activity income.

In the current year, a taxpayer reports the following items: Salary $50,000 Income from partnership A, in which the taxpayer materially participates 20,000 Passive activity loss from partnership B (40,000) During the year, the taxpayer disposed of the interest in partnership B, which had a suspended loss carryover of $10,000 from prior years. What is the taxpayer's adjusted gross income for the current year? (1) $20,000 (2) $30,000 (3) $60,000 (4) $70,000

Choice "1" is correct. The $50,000 salary and $20,000 ordinary business income from active partnership A are fully taxable active income. Generally, passive activity losses (PALs) can only be offset against passive activity income in the current or future years. However, in the year in which a taxpayer disposes of a passive activity, any current or suspended PALs for that activity may be offset against any other sources of income (active, passive, or portfolio). Because the taxpayer disposed of the interest in partnership B during the current year, the current year PAL of $40,000 and the suspended PALs from prior years of $10,000 can be offset against the current year active income. Salary $50,000 Partnership A ordinary business income 20,000 Partnership B ordinary business loss (40,000) Partnership B suspended PALs (10,000) Adjusted gross income $20,000

Wade Inc. granted a nonqualified stock option for 100 shares at $50 per share to Mary, an employee, on May 1, Year 12. On that date, the option was selling on an established market for $4 per share. Mary exercised the option on August 2, Year 13, when the FMV was $80 per share. She sold the stock on September 2, Year 14, for $100 per share. How much gross income and what type did Mary recognize in Year 12? (1) $400 ordinary income (2) $400 capital gain (3) $5,000 ordinary income (4) $5,000 capital gain

Choice "1" is correct. The employee receiving a nonqualified stock option must recognize as ordinary income the value of the option if traded on an established market. Here, that is 100 shares at $4 per share, or $400. Choice "2" is incorrect. This is the correct amount, but it is ordinary income and not a capital gain.

A married couple reported the following items for the current year: Salaries $95000 Dividends 1,000 Interest income on savings account 500 Loss from rental real estate (2,000) Both spouses actively participate in the rental real estate activities. What is the taxpayers' AGI on a joint return for the year? (1) $94500 (2) $95000 (3) $96500 (4) $98500

Choice "1" is correct. The general rule is that losses from passive activities (including rental activities) can only be deducted against income from passive sources. However, one exception to this general rule is that up to $25,000 of net passive losses from the rental of real estate may be deducted against income from nonpassive sources if the taxpayers are actively involved in managing the real estate and the taxpayers' AGI does not exceed $100,000 (a phase-out applies for AGI between $100,000 and $150,000). Because the taxpayers are actively involved in the rental real estate and their AGI is less than $100,000, the entire $2,000 rental real estate loss may be deducted. AGI is computed as follows: Salaries $95,000 Dividends 1,000 Interest income on savings account 500 Loss from rental real estate (2,000) AGI $94,500 Choice "2" is incorrect. The $1,000 of dividends and the $500 of interest income are both included in AGI, and the $2,000 loss from the rental real estate is deductible because the taxpayers are actively involved with the rental real estate and their AGI is less than $100,000. Choice "3" is incorrect. The $2,000 loss from the rental real estate is deductible because the taxpayers are actively involved with the rental real estate and their AGI is less than $100,000. Choice "4" is incorrect. The $2,000 from the rental real estate is a loss, not income, and thus, if deductible, it must be subtracted and not added. Also, the $2,000 loss from the rental real estate is deductible because the taxpayers are actively involved with the rental real estate and their AGI is less than $100,000.

IRC Section 263A requires the capitalization of certain indirect costs related to inventory when a qualifying business is manufacturing tangible personal property. Which of the following costs is not required to be capitalized as part of this adjustment? (1) Marketing (2) Recruiting (3) Payroll (4) Securities services

Choice "1" is correct. The general rule is that product costs are capitalized, such as direct materials, indirect materials, and factory overhead. Period expenses are not capitalized, including G&A, selling, and R&D. Marketing is a period expense that is not capitalized. Choices "2", "3", and "4" are incorrect. These are all factory overhead items related to product costs that would be capitalized.

During a major sports event, a taxpayer rented his primary residence to spectators for 10 days. The taxpayer's rental income and expenses were as follows: Rental income $10,000 Prorated mortgage interest and taxes 1,000 Advertising 500 Commissions 1,000 How much net rental income must the taxpayer report on his income tax return? (1) $0 (2) $7,500 (3) $8,500 (4) $10,000

Choice "1" is correct. The taxpayer rented his primary residence for less than 15 days during the year, so he is not required to include the rental income on his income tax return, nor is he allowed to deduct the commissions and advertising expenses related to the rental activity. He is still allowed to deduct the mortgage interest and real estate taxes as itemized deductions. Choice "2" is incorrect. The net income from rental of the taxpayer's residence, including a prorated portion of mortgage interest and real estate taxes, is $7,500. However, the taxpayer is not required to include the rental income on his income tax return because he rented his residence for less than 15 days during the year. Choice "3" is incorrect. The net income from rental of the taxpayer's residence, excluding the prorated portion of mortgage interest and real estate taxes, is $8,500. However, the taxpayer is not required to include the rental income on his income tax return because he rented his residence for less than 15 days during the year. Choice "4" is incorrect. The taxpayer is not required to include the $10,000 of rental income on his income tax return because he rented his residence for less than 15 days during the year.

For a cash basis taxpayer, gain or loss on a year-end sale of listed stock arises on the: (1) Trade date (2) Settlement date (3) Date of receipt of cash proceeds (4) Date of delivery of stock certificate

Choice "1" is correct. Trade date. Gain or loss on a year-end sale of listed stock arises on the trade date. Rule: Whether on the cash or accrual method of accounting taxpayers who sell stock or securities on an established securities market must recognize gains and losses on the trade date, rather than on the settlement date.

Which of the following costs is not included in inventory under the Uniform Capitalization rules for goods manufactured by the taxpayer? (1) Research (2) Warehousing costs (3) Quality control (4) Taxes excluding income taxes

Choice "1" is correct. Uniform Capitalization rules provide guidelines with respect to capitalizing or expensing certain costs. With regard to inventory, direct materials, direct labor, and factory overhead should be capitalized as part of the cost of inventory. Warehousing costs, quality control, and taxes, excluding income taxes, are all considered factory overhead items. The research should be expensed.

Vale is a 50% partner in Ball Partnership. Vale's tax basis in Ball on January 2, year 1, was $60,000. Ball did not have unrealized receivables, appreciated inventory, or properties that had been contributed by its partners. On December 31, year 1, Ball made a $10,000 non-liquidating cash distribution to each partner. The Ball Partnership income tax return reported the following items for yea: Tax-exempt interest income $80,000 Dividend income 12,000 What total amount of gross income from Ball should be included in Vale's year 1 adjusted gross income? (1) $6,000 (2) $16,000 (3) $36,000 (4) $46,000

Choice "1" is correct. Vale includes only his share of dividend income from the partnership ($12,000 × 50% = $6,000) in adjusted gross income. Choice "2" is incorrect. Adjusted gross income does not include cash distributions if the distributions do not exceed the partner's tax basis in the partnership interest prior to distribution. Choice "3" is incorrect. Adjusted gross income does not include tax-exempt interest income because it is not taxable. Cash distributions decrease the partner's tax basis in the partnership interest, and do not decrease adjusted gross income. Choice "4" is incorrect. Adjusted gross income does not include tax-exempt interest income because it is not taxable.

John and Mary were divorced in 2017. The divorce decree (executed June 30, 2017) provides that John pay alimony of $10,000 per year, to be reduced by 20 percent on their child's 18th birthday. During the current year, the $10,000 was paid in the following way: John paid $7,000 directly to Mary and $3,000 to Spring College for Mary's tuition. What amount of these payments should be reported as income in Mary's current year income tax return? (1) $5,600 (2) $8,000 (3) $8,600 (4) $10,000

Choice "2" is correct. Alimony paid pursuant to a divorce or separation agreement executed before 12/31/18 would be income to Mary while child support would not. Funds qualify as child support only if 1) a specific amount is fixed or is contingent on the child's status (e.g., reaching a certain age); 2) it is paid solely for the support of minor children; and 3) it is payable by decree, instrument, or agreement. The actual use of the funds is irrelevant to the issue. In this case, $2,000 (20% × $10,000) qualifies as child support. The other $8,000 is alimony, which would be income to Mary. Note that for all divorce or separation agreements executed after December 31, 2018, the alimony is neither taxable to the recipient nor deductible by the payor. Choice "1" is incorrect. Take 80% of the $10,000 paid, not 80% of the $7,000 received by Mary. Choice "3" is incorrect. Only $8,000 would be alimony per the divorce decree (80% × $10,000). Choice "4" is incorrect. The 20% reduction when the child turns 18 makes 20% of the $10,000 payment, or $2,000, child support, which is nontaxable to Mary.

The question below includes actual dates that must be used to determine the appropriate tax treatment of the transaction. Fred and Wilma were divorced in 2015. Fred is required to pay Wilma $12,000 of alimony each year until their child turns 18. At that time, the payment will be reduced to $10,000 per year. In 2020, in accordance with the divorce agreement, Fred paid $6,000 directly to Wilma and $6,000 directly to the law school Wilma is attending. What amount of the payments received in 2020 is income to Wilma? (1) $6,000 (2) $10,000 (3) $12,000 (4) $0

Choice "2" is correct. Alimony pursuant to a divorce or separation agreement executed on or before December 31, 2018, is taxable to the recipient and deductible by the payor. Child support is not taxable to the recipient and not deductible by the payor. Because the total payment decreases to $10,000 once Fred and Wilma's child turns 18, the $2,000 decrease is deemed child support. The fact that Fred pays the law school in accordance with the divorce agreement on Wilma's behalf does not change the fact that $10,000 is considered alimony. Choice "1" is incorrect. The fact that Fred pays the law school in accordance with the divorce agreement on Wilma's behalf does not change the fact that $10,000 is considered alimony. Choice "3" is incorrect. Alimony paid in accordance with a divorce or separation agreement executed on or before December 31, 2018, is taxable to the recipient and deductible by the payor. Child support is not taxable to the recipient and not deductible by the payor. Because the total payment decreases to $10,000 once Fred and Wilma's child turns 18, the $2,000 decrease is deemed child support. Choice "4" is incorrect. Alimony paid according to a divorce or separation agreement executed on or before December 31, 2018, is taxable to the recipient and deductible by the payor.

Which of the following is taxable as gross income? (1) Child support received based on a divorce agreement executed in 2015 (2) Alimony received based on a divorce agreement executed in 2015 (3) Child support received based on a divorce agreement executed in 2019 (4) Alimony received based on a divorce agreement executed in 2019

Choice "2" is correct. Alimony received based on a divorce agreement executed on or before December 31, 2018, is taxable as gross income to the recipient. Choice "1" is incorrect. Child support received is not taxable as gross income. Choice "3" is incorrect. Child support received is not taxable as gross income. Choice "4" is incorrect. Alimony received based on a divorced agreement executed after December 31, 2018, is not taxable as gross income to the recipient.

What is the tax treatment of net losses in excess of the at-risk amount for an activity? (1) Any loss in excess of the at-risk amount is suspended and is deductible in the year in which the activity is disposed of in full (2) Any losses in excess of the at-risk amount are suspended and carried forward without expiration and are deductible against income in future years from that activity (3) Any losses in excess of the at-risk amount are deducted currently against income from other activities; the remaining loss, if any, is carried forward without expiration (4) Any losses in excess of the at-risk amount are carried back three years against activities with income and then carried forward for five years

Choice "2" is correct. Any losses in excess of the at-risk amount are suspended and carried forward without expiration and are deductible against income in future years from that activity. The at-risk amount is also referred to as at-risk basis. Note that although in the textbook we discuss this for partnerships, the concept applies to all activities that have flow through income and losses. Choice "1" is incorrect. This is the rule for suspended passive activity losses, not suspended losses due to at-risk limitations. Any losses in excess of the at-risk amount are suspended and carried forward without expiration and are deductible against income in future years from that activity. Choice "3" is incorrect. Losses in excess of the at-risk amount may not be deducted currently against income from other activities. Any losses in excess of the at-risk amount are suspended and carried forward without expiration and are deductible against income in future years from that activity. Choice "4" is incorrect. Losses in excess of the at-risk amount are not carried back three years against activities with income and then carried forward for five years. These are the carryback/carryforward periods for C corporation capital losses. Any losses in excess of the at-risk amount are suspended and carried forward without expiration until the taxpayer generates more at-risk basis or until the activity is sold.

Baker, a sole proprietor CPA, has several clients that do business in Spain. While on a four-week vacation in Spain, Baker took a five-day seminar on Spanish business practices that cost $700. Baker's round-trip airfare to Spain was $600. While in Spain, Baker spent an average of $100 per day on accommodations, local travel, and other incidental expenses, for total expenses of $2,800. What amount of total expense can Baker deduct on Form 1040 Schedule C, "Profit or Loss From Business," related to this situation? (1) $700 (2) $1,200 (3) $1,800 (4) $4,100

Choice "2" is correct. Baker can deduct $1,200 in total expense on Form 1040 Schedule C, calculated as follows: Direct educational expenses 700 [cost of the course] Daily expenses for 5-day seminar 500 [$100 per day × 5] Total educational expenses 1,200 Rule: If foreign travel is primarily personal in nature (e.g., a vacation), none of the travel expenses (e.g., round-trip airfare) incurred will be allowable business deductions, even if the taxpayer was involved in business activities while in the foreign country. Choice "1" is incorrect, as the expenses for the five-day period Baker attended the seminar were directly related to being in Spain for the additional period of time and are allowable business deductions.

The Groves own a beach house as a second home. This year, the Groves used the beach house personally for 4 months. For 14 days during the summer, the Groves rented out their beach house for $5,000 total to friends. Which statement is true regarding the taxability of the Groves' beach house? (1) 5,000 is included in gross income (2) Mortgage interest paid on the beach house is deductible (3) All repair expenses on the beach house are deductible (4) Depreciation expense on the beach house is deductible.

Choice "2" is correct. Because the Groves rented their cabin for fewer than 15 days, it is treated as a personal residence. Therefore, the rental income is excluded from gross income and mortgage interest and real estate taxes are deductible as itemized deductions on schedule A (subject to limitations). Choice "1" is incorrect. Because the Groves rented their cabin for fewer than 15 days, it is treated as a personal residence. Mortgage interest and real estate taxes are deductible as itemized deductions on schedule A (subject to limitations). Choices "3" and "4" are incorrect. Because the Groves rented their cabin for fewer than 15 days, it is treated as a personal residence. Therefore, repairs, utilities, depreciation, and other allowed rental expenses are not deductible.

Anderson, a computer engineer, and spouse, who is unemployed, provide more than half of the support of their child, age 23, who is a full-time student who earns $7,000. They also provide more than half of the support for their older child, age 33, who earns $2,000 during the year. How many dependents meet qualifying relative or qualifying child rules for the Andersons? (1) One (2) Two (3) Three (4) Zero

Choice "2" is correct. Both children meet the dependency criteria. The 23-year-old child meets qualifying child rules (CARES). The age limit is met because the 23-year-old is a full-time student. The 33-year-old meets qualifying relative rules (SUPORT). The Andersons have two dependents for tax purposes.

Dr. Merry, a self-employed dentist, incurred the following expenses: Investment expenses $700 Custodial fees related to Dr. Merry's Keogh plan $40 Work uniforms for Dr. Merry and Dr. Merry's employees $320 Subscriptions for periodicals used in the waiting room $110 Dental education seminar $1,300 What is the amount of expenses the doctor can deduct as business expenses on Schedule C, Profit or Loss from Business? (1) $1,620 (2) $1,730 (3) $1,770 (4) $2,430

Choice "2" is correct. Business expenses include work uniforms for the taxpayer and taxpayer's employees, subscriptions for periodicals for patient use, and continuing education expenses. Choice "1" is incorrect. Business expenses include subscriptions for periodicals for patient use. Choice "3" is incorrect. Business expenses do not include custodial fees for retirement accounts. Choice "4" is incorrect. Business expenses do not include investment expenses

On December 1 of the current taxable year, Krest, a self-employed cash basis taxpayer, borrowed $200,000 to use in her business. The loan was to be repaid on November 30 of the following year. Krest paid the entire interest amount of $24,000 on December 1 of the current year. What amount of interest was deductible on Krest's current year income tax return? (1) $0 (2) $2,000 (3) $22,000 (4) $24,000

Choice "2" is correct. Cash basis taxpayers deduct interest in the year paid or the year to which the interest relates, whichever is later. Even though all of the interest on this loan was paid on December 1, of the current year, only the interest relating to December of the current year can be deducted in the current year. The question does not give an interest rate, but because the loan is to be repaid in a lump sum at maturity, 1/12 of the interest, or $2,000 applies to each month. Choice "1" is incorrect. Because $2,000 of the interest relates to the current year, this amount is deductible in the current year. Choice "3" is incorrect. This is the amount that cannot be deducted until the following year, the year to which the interest relates. Be sure to read questions like this very carefully, because if you had simply misread the question as seeking the amount deductible in the following year, you would get the question wrong despite understanding the rule. Choice "4" is incorrect. Cash basis taxpayers can deduct interest in the year paid or the year to which the interest relates, whichever is later, thus 11 months of the interest will not be deductible until next year.

Miyasyke, Inc., a calendar year S corporation, has 5 equal shareholders at the end of the tax year. Miyasyke had $75,000 of taxable income. Miyasyke made distributions to its shareholders of $32,000 each, for a total of $160,000. Each shareholder's basis in the S corporation is $100,000 at the beginning of the tax year. What amount from Miyasyke should be included in each shareholder's gross income? (1) $0 (2) $15,000 (3) $32,000 (4) $47,000

Choice "2" is correct. Each shareholder reports his/her pro rata share of the S corporation's taxable income in his or her gross income. The distributions are not taxable to the extent the shareholders' basis exceeds the distribution (and increased for any income reported by them during the year). Choice "1" is incorrect. Each shareholder's share of taxable income (non-separately stated) is reported in the shareholder's gross income. Choices "3" and "4" are incorrect. Choice "3" only includes the distribution, which is not taxable in this case as the shareholder's basis exceeds the distribution. Choice "4" includes both the shareholder's pro rata share of the taxable income and the distribution. The distribution is not taxable in this situation.

Juan recently started operating a flower shop as a proprietorship. In its first year of operations, the shop had a taxable income of $60,000. Assuming that Juan had no other employment-related earnings: (1) The flower shop must withhold FICA taxes from Juan's earnings (2) Juan must pay self-employment tax on the earnings of the business (3) Juan will be exempt from self-employment taxes for the first three years of operations (4) Juan will be exempt from the Medicare tax because the business earnings are below the threshold amount

Choice "2" is correct. Earnings from self-employment are subject to the income tax as well as to the federal self-employment tax. Thus, Juan must pay self-employment tax on the earnings of the business. Choice "1" is incorrect. Because Juan is the proprietor of a sole proprietorship (as opposed to an employee of an employer), the flower shop will not pay Juan a wage, and thus will not withhold either income taxes or FICA taxes from that wage. Choice "3" is incorrect. There is no provision exempting a proprietorship from self-employment taxes for its first three years of operations. Choice "4" is incorrect. No self-employment tax is owed if self-employment income, after multiplying by 92.35 percent, is less than $400. However, Juan's self-employment income exceeds this threshold, and thus Juan is not exempt from either of the two components of the self-employment tax (the Medicare tax and the Social Security tax).

Clark did not itemize deductions on his Year 8 federal income tax return. In July Year 9, Clark received a state income tax refund of $900 plus interest of $10, for overpayment of Year 8 state income tax. What amount of the state tax refund and interest is taxable on Clark's Year 9 federal income tax return? (1) $0 (2) $10 (3) $900 (4) $910

Choice "2" is correct. Except for interest from state and local government bonds, interest income is fully taxable, so the $10 is included in income. Since Clark did not itemize deductions on his Year 8 federal income tax return, he did not deduct any state income taxes last year. Under the tax benefit rule, the refund is not taxable this year because Clark did not deduct the tax last year.

David is a CPA and enjoys playing the lottery. This year, David won $10,000 in lottery scratch-off tickets. He spent $200 purchasing the tickets. Which statement is true regarding David's winnings? (1) David must include $9,800 in gross income (2) David must include $10,000 in gross income and can deduct $200 as an itemized deduction (3) David must include $10,000 in gross income and can deduct $200 as an adjustment to AGI (4) David's winnings are not taxable

Choice "2" is correct. Gambling winnings are included in gross income. Gambling losses are deductible to the extent of gambling winnings, but are deducted on Schedule A and not calculated as part of gross income.

Gena, an unmarried individual, had an adjusted gross income of $125,000 in the current year before any IRA deduction, taxable social security benefits, or passive activity losses. Gena incurred a loss of $30,000 in the current year from rental real estate in which she actively participated. What amount of loss attributable to this rental real estate can be used in the current year as an offset against her income from non-passive sources? (1) $0 (2) $12,500 (3) $15,000 (4) $25,000

Choice "2" is correct. Gena may use $12,500 of the loss attributable to her rental real estate activities as an offset to her income from non-passive sources in the current year. RULE: Rental real estate activities are passive activities, and losses from them are generally not allowed to be used as an offset against income from any non-passive activities. However, there is a limited exception to this general rule in the case where a taxpayer actively participates in rental real estate. Under this exception, up to $25,000 of passive losses may be used to offset income from non-passive sources. This $25,000 allowance is reduced (not below zero) by an amount equal to 50% of the amount by which the taxpayer's modified AGI exceeds $100,000 (becoming fully phased-out at modified AGI of $150,000). In this case, modified AGI of $125,000 is $25,000 higher than the $100,000 floor. The allowance of the $25,000 exception (which would apply in Gena's case) is reduced by 50% of the difference (or $12,500). Therefore, the amount allowable to be used to offset against non-passive sources is $12,500. Note that MFS filers are not allowed any loss deduction amount unless they lived apart the entire year. If MFS filers do live apart for the entire year, they each can claim a maximum deduction of $12,500 before the phase-out, which begins when MAGI exceeds $50,000. Choice "1" is incorrect. Gena actively participates in the rental real estate activity and her modified AGI does not exceed $150,000, therefore, she is entitled to a deduction. Choice "3" is incorrect. This answer represents 50% of the loss from rental real estate activities, which is not the proper calculation. Choice "4" is incorrect. This is the entire loss from rental real estate activities, which is limited based upon Gena's modified AGI.

Which of the following statements is not correct? (1) Employee stock purchase plans are a type of qualified stock option plan (2) The receipt of an incentive stock option with generally have to report compensation income in the year that the option is received (3)The employer may recognize a deductible expense for a nonqualified stock option in the same year that the employee will recognize ordinary income (4) For an incentive stock option, once exercised, the stock must be held for at least two years after the grant date and at least one year after the exercise date

Choice "2" is correct. Generally there is no recognition of compensation expense with an incentive stock option. Choices "1", "3", and "4" are incorrect, as these are all true statements.

Mr. and Mrs. Williams decided during the tax year to purchase their first new home. The fair market value of the home was $275,000, and a 20 percent down payment was required to secure a mortgage in the amount of $220,000 at 5 percent for 30 years. The Williams' decided to utilize $10,000 that was kept in an Individual Retirement Account owned by Mrs. Williams. This amount was withdrawn on June 12 and used to fund the down payment on July 1. These amounts had been previously deducted as an adjustment by her on an individual tax return in the year of contribution. The remaining $12,000 for the down payment was drawn from a savings account. How much of the distribution from the Individual Retirement Account is subject to the premature distribution penalty tax, and how much must be included in the Williams' joint tax return in the year of distribution as gross income? (1) Penalty Tax $0, Gross Income $0 (2) Penalty Tax $0, Gross Income $10,000 (3) Penalty Tax $10,000, Gross Income $0 (4) Penalty Tax $10,000, Gross Income $10,000

Choice "2" is correct. Generally, a premature distribution (prior to retirement or other allowable age) from an individual retirement account is subject to a 10 percent penalty tax. Certain exceptions to this tax are available and are contained in the mnemonic "HIM DEAD." Home buyer (1st time) $10,000 max if used toward first home Insurance (medical) Medical expenses in excess of percentage of AGI floor Disability Education Adoption or birth of child made within one year from the date of birth or adoption ($5,000 maximum exclusion) Death The amount removed from the IRA qualifies under the "H" exception above. However, the question states that these amounts had been previously deducted on Mrs. Williams' individual tax return, thus this is a distribution from a traditional, deductible IRA. When distributed, funds held in a traditional, previously deducted IRA are taxable to the recipient as ordinary income and thus would be included as gross income on the Williams' joint tax return in the year of distribution. Choice "1" is incorrect. The distribution would be included in the Williams' gross income. Choice "3" is incorrect. The amount qualifies for an exception to the penalty tax and would be included in the Williams' gross income. Choice "4" is incorrect. The amount qualifies for an exception to the premature distribution penalty tax.

The Jacksons, who file a joint return, actively participate in a solely-owned rental real estate activity that produces a $30,000 loss during the current year. Their adjusted gross income was $120,000 before considering the rental activity. How much of the rental loss, if any, are the Jacksons entitled to deduct? (1) $0 (2) $15,000 (3)$25,000 (4) $30,000

Choice "2" is correct. Generally, passive losses are only deductible against other passive income, and there is no passive income in the facts of this question. However, the "mom and pop" exception will apply because the Jacksons actively participate in the activity. This exception allows up to $25,000 of passive losses to be deducted against other nonpassive income. There is a phase-out over an adjusted gross income (AGI) range of $100,000 to $150,000. The Jacksons' AGI is $120,000, and that is 40% into the phase-out range. Therefore, 40% of the $25,000 exception amount is phased out, and the deduction is $15,000 [$25,000 - ($25,000 × 40%)]. Choice "1" is incorrect. $0 would be correct if the "mom and pop" exception did not apply or was completely phased out. Choice "3" is incorrect. $25,000 would be the correct answer if the entire "mom and pop" exception applied. Choice "4" is incorrect. $30,000 would only be correct if at least that amount of other passive income existed.

James Corp. issue stock options to employees under an Employee Stock Purchase Plan. Which of the following statements is correct? I.The option exercise price must be less than the lesser of 95% of the FMV of the stock when granted or exercised. II.The option cannot be exercised more than 27 months after the grant date. (1) I only (2) II only (3) Both (4) Neither

Choice "2" is correct. I is not correct because the rule states 85%, not 95%. II is a correct statement. This is a requirement of an ESPP.

Mary purchased an annuity that pays her $500 per month for the rest of her life. She paid $70,000 for the annuity. Based on IRS annuity tables, Mary's life expectancy is16 years. How much of the first $500 payment will Mary include in her gross income (round to two decimals)? (1) Deduct $43,749.60 as a miscellaneous itemized deduction not subject to 2% of AGI floor (2) Deduct $26,250.40 as a miscellaneous itemized deduction not subject to 2% of AGI floor (3) Deduct $70,000 as a miscellaneous itemized deduction not subject to 2% of AGI floor (4) No deduction for the annuity

Choice "2" is correct. If Mary dies after receiving 10 full years of annuity payments, she will have received 120 payments (10 years x 12 months). Mary's IRS life expectancy was 16 years (16 years x 12 months = 192 months). For the first 192 payments, Mary will have a return of capital of $364.58 ($70,000/192 months). Therefore, after 10 full years of payments she will have recovered $43,749.60 ($364.58 x 12 x 10) of the $70,000 investment in the annuity. The unrecovered portion ($70,000 - $43,749.60 = $26,250.40) can be deducted on Mary's final tax return as a miscellaneous itemized deduction not subject to the 2% of AGI floor.

A taxpayer received $200 in interest from U.S. Treasury bonds and $300 in interest from municipal bonds. What amount of interest should be included in the taxpayer's gross income? (1) $0 (2) $200 (3) $300 (4) $500

Choice "2" is correct. In general, all income from whatever source derived is included in gross income. However, interest from state and local government bonds (i.e., "municipal" bonds) is not included in gross income. It is important to note, however, that the U.S. government is not a municipality; thus, U.S. obligations such as Treasury bonds are not municipal bonds and therefore interest on such obligations is included in gross income. Choice "1" is incorrect. Interest on municipal bonds is tax exempt, but the U.S. government is not a municipality; thus, U.S. obligations such as Treasury bonds are not municipal bonds and therefore interest on such obligations is included in gross income. Choice "3" is incorrect. Interest from municipal bonds is tax exempt; therefore, the $300 of interest on the municipal bond is not included in the taxpayer's gross income. The U.S. government is not a municipality; thus, U.S. obligations such as Treasury bonds are not municipal bonds, and therefore the $200 of interest on the U.S. Treasury bond is included in the taxpayer's gross income. Choice "4" is incorrect. Interest from municipal bonds is tax-exempt; therefore, the $300 in this problem is not included in the taxpayer's gross income.

Pat has various items of income as follows: W-2 wages $24,000 Interest and dividends $3,000 Sole proprietorship income on Schedule C $8,000 Income from an S corporation from Schedule K-1 $12,000 Income from a general partnership from Schedule K-1 $10,000 For purposes of the self-employment tax, what are the net earnings from self-employment? (Note: Please answer before the required 92.35% calculation performed on Schedule SE.) (1) $10,000 (2) $18,000 (3) $22,000 (4) $57,000

Choice "2" is correct. Income subject to self-employment includes amounts from an unincorporated sole proprietorship (Schedule C) and general partnerships. It does not include W-2 wages, interest, dividends, or income from an S corporation. The only amounts here that qualify are the $8,000 sole proprietorship income and the $10,000 from the general partnership.

Danny received the following interest and dividend payments this year. What amount should Danny include in his gross income? City of Atlanta bond interest $1,200 U.S. Treasury bond interest 500 State of Georgia bond interest 1,000 Ellis Company common stock dividend 400 Row Corporation bond interest 600 (1) $2,500 (2) $1,500 (3) $3,700 (4) $2,200

Choice "2" is correct. Interest on municipal bonds (bonds issued by state or local governments) is excluded from gross income. Therefore, the city of Atlanta and State of Georgia bond interest is not taxable. The U.S. Treasury and Row Corporation bond interest is taxable. The Ellis Company stock dividend is also taxable. Choices "1" and "3" are incorrect. Interest on municipal bonds (bonds issued by state or local governments) is excluded from gross income. Therefore, the city of Atlanta and State of Georgia bond interest is not taxable. Choice "4" is incorrect. Although interest on municipal bonds (bonds issued by state or local governments) is excluded from gross income, income from interest on U.S. Treasury bonds and interest and dividends from corporations are included in taxable income.

Nan, a cash basis taxpayer, borrowed money from a bank and signed a 10-year interest-bearing note on business property on January 1 of the current year. The cash flow from Nan's business enabled Naan to prepay the first three years of interest attributable to the note on December 31 of the current year. How should Nan treat the prepayment of interest for tax purposes? (1) Deduct the entire amount as a current expense (2) Deduct the current year's interest and amortize the balance over the next two years (3) Capitalize the interest and amortize the balance over the 10-year long period (4) Capitalize the interest as part of the basis of the business property

Choice "2" is correct. Interest paid in advance by a cash basis taxpayer on business loans cannot be deducted until the tax period to which the interest relates. In other words, the interest must be both paid and incurred in order to be deducted.

John and Theresa are in the process of obtaining a divorce. Although they are not legally separated, John moved out of the family home in October of Year 1 and moved into an apartment nearby. John and Theresa's two children, Jenna and Stella, lived with Theresa in the family home for more than half of the tax year. What filing status can Theresa use to file her Year 1 tax return? (1) Head of household (2) Married filing jointly/separately (3) Surviving spouse (qualifying widow) (4) Single

Choice "2" is correct. John and Theresa are still married at year-end, not legally separated, and have not lived apart for the last six months of the taxable year. Theresa must file as married, but may choose to do so either jointly with John or separately. Choice "1" is incorrect. Head of household status is not an option because the couple is not legally separated at year-end and John did not live apart from Theresa for the last six months of the taxable year. Choice "3" is incorrect. Surviving spouse (qualifying widow) is not an option for Theresa, as John is still alive. Choice "4" is incorrect. Filing as single is not an option, because John and Theresa are still married and not legally separated at year-end.

On February 1 of Year 0, John received a nonqualified stock option to purchase 100 shares of his employer's stock for $10 per share. At the time John received the option, it was selling for $5 per share on an established exchange. On September 1 of Year 1, John exercised the options when the stock was selling for $19 per share. On December 1 of Year 2, John sold all of the shares for $30 per share. What is the amount and character of income that John must report in Year 0? (1) $0 (2) $500, ordinary (3) $500, capital (4) $1,000, ordinary

Choice "2" is correct. John must recognize $500 (100 shares x $5/option) of ordinary income (equivalent to compensation) on the date the options were granted because the option has a readily ascertainable market value.

Mort and Mindy met at a New Year's Eve party held December 31, Year 1. They instantly bonded, fell madly in love, and were married at 11:38 p.m that night. Sadly, Mort passed away November 15, Year 2. What filing status should Mindy use for Year 2? (1) Single (2) Married filing jointly (3) Head of household (4) Surviving spouse

Choice "2" is correct. Mindy will be able to use the married filing jointly status for the year Mort passed away (Year 2) even though she was not married at year-end.

Which of the following is the overall limitation to the qualifying business income (QBI) deduction? (1) Lesser of: 50 percent of combined QBI or 20 percent of the taxpayer's taxable income in excess of net capital gain (2) Lesser of: combined QBI or 20 percent of the taxpayer's taxable income in excess of net capital gain (3) Lesser of: 50 percent of W-2 wages or 25 percent of W-2 wages plus 2.5 percent of the unadjusted basis of qualified property (4) Taxable income limitations based on filing status

Choice "2" is correct. Once the QBI deduction is calculated based on the taxpayer's eligibility, the overall deduction is limited to the lesser of combined QBI or 20 percent of the taxpayer's taxable income in excess of net capital gain. Choice "1" is incorrect. Once the QBI deduction is calculated based on the taxpayer's eligibility, the overall deduction is limited to the lesser of combined QBI (not 50 percent of the combined QBI) or 20 percent of the taxpayer's taxable income in excess of net capital gain. Choice "3" is incorrect. The wage and property limitation determines the calculation of the QBI deduction but is not the overall limitation to the QBI deduction. Choice "4" is incorrect. Taxable income limitations based on filing status determine the calculation of the QBI deduction. The overall limitation to the deduction, however, is the lesser of combined QBI or 20 percent of the taxpayer's taxable income in excess of net capital gain.

Jim and Kay Ross contributed to the support of their two children, Dale and Kim, and Jim's widowed parent, Grant. For Year 27, Dale, a 19-year-old full-time college student, earned $4,500 as a babysitter. Kim, a 23-year-old bank teller, earned $12,000. Grant received $5,000 in dividend income and $4,000 in nontaxable Social Security benefits. Grant and Kim are U.S. citizens and were over one-half supported by Jim and Kay, but neither of the two currently reside with Jim and Kay. Dale's main place of residence is with Jim and Kay, and he is currently on a temporary absence to attend school. How many people meet the definition of either qualifying child or qualifying relative on the Year 27 joint income tax return for Jim and Kay Ross? (1) Zero (2) One (3) Two (4) Three

Choice "2" is correct. Only one person meets the criteria for either qualifying child or relative for the Rosses. Dale meets the definition of qualifying child. He meets all criteria of CARES. He is under the age limit because he is a full-time student under the age of 24. All other CARES tests are met. Kim does not meet the age test for qualifying child. She also does not meet the qualifying relative criteria. She fails the gross income limitations of SUPORT. Choice "1" is incorrect. Dale does meet the criteria for qualifying child for Jim and Kay Ross. Choice "3" is incorrect. Kim fails the age test for qualifying child (CARES) and is not under the gross income limitation for qualifying relative (SUPORT). Choice "4" is incorrect. Only Dale meets the definition of either qualifying child or relative for Jim and Kay Ross. Kim does not meet the age limit for QC or the gross income limitation for QR.

Red Inc. provides group term life insurance to the employees of the corporation. Susan, a manager, received $200,000 of coverage for the year at a cost to Red, Inc. of $2,800. The Uniform Premiums (based on Susan's age) are $9 a year for $1,000 protection. How much of the premiums must Susan include in gross income this year? (1) $0 (2) $1,350 (3) $1,800 (4) $2,800

Choice "2" is correct. Premiums for coverage in excess of $50,000 of coverage are taxable to the employee. Total coverage 200,000 Maximum nontaxable coverage (50,000) Excess taxable 150,000 Units ÷ 1,000 Taxable units 150 Taxable cost per unit × 9 Taxable benefit 1,350 Choice "1" is incorrect. Only the premiums on $50,000 of coverage are nontaxable to the employee. Choice "3" is incorrect. Only the premiums on the coverage in excess of $50,000 are taxable. Choice "4" is incorrect. Premiums on the first $50,000 of coverage are nontaxable to the employee, so the entire premium of $2,800 could not be the correct answer.

On December 1 of the prior year, Michaels, a self-employed cash basis taxpayer, borrowed $100,000 to use in her business. The loan was to be repaid on November 30 of the current year. Michaels paid the entire interest of $12,000 on December 1 of the prior year. What amount of interest was deductible on Michaels' current year income tax return? (1) $12,000 (2) $11,000 (3) $1,000 (4) $0

Choice "2" is correct. Prepaid interest must be prorated over the time for which payment is made. This is true for both cash and accrual basis taxpayers. The loan is for 1 month in Year 1 and 11 months in Year 2. Therefore, 1/12 of the interest is deductible in Year 1 and 11/12, or $11,000 is deductible in Year 2. Choices "1", "3", and "4" are incorrect. Prepaid interest must be prorated over the time for which payment is made. This is true for both cash and accrual basis taxpayers.

Klein, a master's degree candidate at Blair University, was awarded a $12,000 scholarship in year 8. The scholarship was used to pay Klein's year 8 university tuition and fees. Also in year 8, Klein received $5,000 for teaching two courses at a nearby college. What amount is includable in Klein's year 8 gross income? (1) $0 (2) $5,000 (3) $12,000 (4) $17,000

Choice "2" is correct. Scholarships are nontaxable for degree-seeking students to the extent that the proceeds are spent on tuition, fees, books, and supplies. The $5,000 for teaching courses is taxable compensation for services delivered. Choice "1" is incorrect. The $5,000 for teaching courses is taxable compensation for services delivered. Choice "3" is incorrect. The scholarship is not taxable because Klein is a degree-seeking student and used the proceeds for tuition and fees. Furthermore, the $5,000 for teaching courses is taxable compensation for services delivered. Choice "4" is incorrect. The scholarship is not taxable because Klein is a degree-seeking student and used the proceeds for tuition and fees.

Which of the following statements regarding the self-employment tax is true? (1) Income and expenses from self-employment is reported on Schedule D (Form 1040) (2) Self-employment income is subject to both federal income tax and self-employment tax (3) One half of self-employment tax is deductible as an itemized deduction (4) All self-employment income is subject to both Medicare and Social Security tax

Choice "2" is correct. Self-employment income is subject to federal income tax and self-employment tax. The self-employment tax is made up of Social Security (12.4%) and Medicare (2.9%), for a total of 15.3%. Choice "1" is incorrect. Income from self-employment is reported on Schedule C of Form 1040. Choice "3" is incorrect. One half of self-employment tax is deductible as an AGI deduction. Choice "4" is incorrect. All self-employment income is subject to the 2.9% Medicare tax, but Social Security tax is subject to the 12.4% tax only up to a certain threshold.

Which of the following statements regarding an individual's suspended passive activity losses is correct? (1) $3,000 of suspended losses can be utilized each year against portfolio income (2) Suspended losses can be carried forward, but not back, until utilized (3) Suspended losses must be carried back three years and forward five years (4) A maximum of 50 percent of the suspended losses can be used each year when an election is made to forgo the carryback period

Choice "2" is correct. Tax rules allow suspended passive losses to be carried forward, but not back, until utilized. Choice "1" is incorrect. This is the rule for capital losses. It does not apply to passive losses. Choice "3" is incorrect. This rule is not correct. There is no carryback allowed for suspended passive losses. This is the carryback/carryforward period for C corporation capital losses. Choice "4" is incorrect. This rule is not correct. There is no carryback allowed for suspended passive losses.

Stephen is a graduate student at West University. He works part time at the campus coffee shop earning $5,000 this year. Stephen also receives a $25,000 scholarship that pays for his tuition, fees, and books. What amount does Stephen include in his gross income? (1) $25,000 (2) $5,000 (3) $30,000 (4) $0

Choice "2" is correct. The $5,000 earnings from the part-time job is taxable because this is compensation. The $25,000 scholarship is not taxable. Scholarships used for tuition, fees, and books are not included in gross income. Choice "1" is incorrect. The $25,000 scholarship is not taxable. Scholarships used for tuition, fees, and books are not included in gross income, but compensation received is taxable income. Choice "3" is incorrect. The $5,000 earnings from the part-time job is taxable because this is compensation. The $25,000 scholarship is not taxable as it is used to paying qualifying expenses. Scholarships used for tuition, fees, and books are not included in gross income. Choice "4" is incorrect. Although the $25,000 scholarship may be excluded from income, the $5,000 earnings from the part-time job is taxable compensation.

A retiree invested $100,000 in an annuity that pays $12,000 annually for 10 years. What portion of the first payment should be included in the retiree's gross income? (1) $0 (2) $2,000 (3) $10,000 (4) $12,000

Choice "2" is correct. The annuity contract is a fixed-period annuity with payments received over 10 years. The original investment in the annuity contract is $100,000 so $10,000 of each annuity payment is nontaxable return of capital ($100,000 / 10 years). The annual annuity payment is $12,000, so $2,000 is included in gross income ($12,000 payment − $10,000 return of capital). Choice "1" is incorrect. The taxpayer must include $2,000 of the first payment, the excess over the $10,000 return of capital, in gross income. Choice "3" is incorrect. The $10,000 is nontaxable return of capital. The remaining $2,000 of the $12,000 annual payment is included in gross income. Choice "4" is incorrect. Only $2,000 of the $12,000 is included in gross income. The other $10,000 is nontaxable return of capital.

Molly Morris is 15 years old. Molly's parents (James and Beth) divorced in May of the current tax year. Molly lived with both parents until the divorce. Molly does not provide more than half of her own support. James' AGI is $40,000 and Beth's AGI is $35,000. Molly's parents cannot decide who cannot decide who can claim Molly as a dependent for tax purposes. Assuming neither parent waives their right to claim Molly as a dependent, which statement is true? (1) Both parents may claim Molly as a dependent because she lives equal time with each parent (2) James may claim Molly as a dependent because his AGI is higher (3) Beth may claim Molly as a dependent because her AGI is lower (4) Beth and James must alternate claiming Molly as a dependent

Choice "2" is correct. The parent with custody of the child for the greater part of the year may claim the child as a dependent (determined by time, not the divorce decree). Because Molly's parents have equal custody during the year, the parent with the higher AGI would be eligible to claim Molly as a dependent, which in this case would be James. However, James could waive the right to claim Molly as a dependent.

Tom and Sharlene had the following items of income and expense during the taxable year: Self-Employment Activity Gross Income $35,000 Business license fees 500 Marketing Expenses 2,000 Salary paid to Sharlene 10,000 Tom's wages from his job 67,000 Interest from money market 1,500 Gain from sale of securities owned for 3 months 15,000 What is Tom & Sharlene's gross income before adjustments? (1) $106,000 (2) $116,000 (3) $128,500 (4) $131,500

Choice "2" is correct. Tom & Sharlene's gross income is calculated as follows: Net self-employment income 32,500 Tom's wages 67,000 Interest 1,500 Gain from sale 15,000 Total gross Income 116,000 Note: Sharlene's salary is not included as income as 100% of the net self-employment activity is taxable to her. Her salary is considered a draw and is not an allowable business deduction against the gross income of the self-employment activity.

The Morgan Trust, a complex trust, had distributable net income (DNI) in Year 4 of $10,000. Of the $10,000 of DNI, $4,000 was distributed to trust beneficiaries. Of the $4,000 distributed, which taxpayer(s), if any, are responsible for the tax liability on the $4,000 distribution? (1) The Morgan Trust (2) The trust beneficiaries (3) The Morgan Trust and the trust beneficiaries (4) Neither the Morgan Trust nor the trust beneficiaries

Choice "2" is correct. Trusts are separate income tax-paying entities. Distributions made by trusts are deductible by the trust, but taxable to the recipient. This avoids double taxation of trust income. Choice "1" is incorrect. Distributions by trusts of income are deductible by the trust but taxable to the recipient. Choice "3" is incorrect. Double taxation is avoided on trust income due to the distribution deduction for trust income paid to trust beneficiaries. Choice "4" is incorrect. All income in a trust is subject to taxation. Income retained in the trust will be taxed at the trust level only, while income distributed by the trust will be deductible at the trust level, but taxable to the beneficiaries.

Susie, John, Luke, and Will provide support for their 80-year-old mother, Joyce. Joyce lives by herself in an apartment in Miami, Florida. Joyce earned $5,000 this year working at her church. Joyce provides 5% of her own support. Susie provides 30% of Joyce's support, John provides 10% of Joyce's support, Luke provides 15% of Joyce's support, and Will provides 40% of Joyce's support. Under a multiple support agreement, who may claim Joyce as an dependent? (1) Susie, Luke, John, and Will (2) Susie, Luke, and Will (3) Susie and Will (4) Will

Choice "2" is correct. Under a multiple support agreement, Susie, Luke, and Will are eligible to claim Joyce as a dependent because they contributed more than 10% of Joyce's support. Choice "1" is incorrect. John did not provide more than 10% of Joyce's support. Therefore, he is not eligible to claim Joyce as a dependent under a multiple support agreement. Choice "3" is incorrect. Under a multiple support agreement, Luke is also eligible to claim Joyce as a dependent because he contributed more than 10% of Joyce's support. Choice "4" is incorrect. Under a multiple support agreement, Susie and Luke are also eligible to claim Joyce as a dependent because they contributed more than 10% of Joyce's support.

Sanderson has made deductible contributions to his traditional IRA for many years. Sanderson recently retired at age 60 and received a distribution of $150,000. In which way, if any, will the distribution be taxed? (1) As a capital gain. (2) As ordinary income. (3) Subject to a 10% penalty. (4) It will not be taxed.

Choice "2" is correct. Withdrawals from traditional IRAs (i.e., IRAs for which the contributions were deducted) are taxed as ordinary income. Withdrawals prior to age 59½ are also subject to a 10 percent penalty tax (unless an exception applies). Because Sanderson is over 59½, the withdrawal is not subject to the 10 percent penalty tax. Choice "1" is incorrect. Withdrawals from traditional IRAs are taxed as ordinary income, not capital gains. Choice "3" is incorrect. Withdrawals from traditional IRAs prior to age 59½ are subject to a 10 percent penalty tax (unless an exception applies). Because Sanderson is over 59½, the withdrawal is not subject to the 10 percent penalty tax. Choice "4" is incorrect. Although withdrawals from Roth IRAs are not taxed provided certain rules are met, withdrawals from traditional IRAs are always taxed.

Angela, a real estate broker, had the following income and expenses in her schedule C business: Commissions income $100,000 Expenses: Commissions paid to non-brokers for referrals (illegal under state law and subject to criminal penalties) 20,000 Commissions paid to other real estate brokers for referrals (not illegal under state law) 10,000 Travel and transportation 12,000 Supplies 4,000 Office and phone 5,000 Parking tickets 500 How much net income must Angela report from this business? (1) $48,500 (2) $49,000 (3) $69,000 (4) $79,000

Choice "3" is correct. Commission income 100,000 Less: Legal commissions 10,000 Less: Travel 12,000 Less: Supplies 4,000 Less: Office and phone 5,000 Total expenses (31,000) Net income 69,000 Choice "1" is incorrect. Illegal payments ($20,000) and fines and penalties ($500) are not deductible expenses on Schedule C. Choice "2" is incorrect. Illegal payments ($20,000) are not deductible expenses on Schedule C. Choice "4" is incorrect. Legal commissions are deductible on Schedule C.

Which payment(s) is (are) included in a recipient's gross income? I. Payment to a graduate assistant for a part-time teaching assignment at a university. Teaching is not a requirement toward obtaining the degree. II. A grant to a Ph.D. candidate for his participation in a university-project for the benefit of the university (1) I only (2) II only (3) Both I and II (4) Neither I nor II

Choice "3" is correct. I. A payment to a student for a part-time teaching assignment is taxable income just as a payment for any other campus job would be. This is not a scholarship or fellowship. II. There is no exclusion in the tax law for amounts paid to a degree candidate for participation in university-sponsored research.

Under a $150,000 insurance policy on her deceased father's life, May Green is to receive $12,000 per year for 15 years. Of the $12,000 received in the current year, the amount subject to income tax is: (1) $0 (2) $1,000 (3) $2,000 (4) $12,000

Choice "3" is correct. $2,000. Death benefit 150,000 Amount received in the current year 12,000 Less: Return of principal ($150,000 ÷ 15 years) (10,000) Taxable interest 2,000

Which of the following is both an item that is an allowable tax deduction to the partnership, reported separately on the individual partner's Schedule K-1, and then included on the partner's individual tax return? (1) Salaries paid to non-partner employees (2) Advertising expeditures (3) Guaranteed payments paid to partners (4) Depreciation on equipment used in the business

Choice "3" is correct. A partnership calculates net ordinary business income or loss and passes each partner's distributive share through on Schedule K-1. Guaranteed payments paid to partners for services provided or for the use of capital, without regard to partnership income or profit and loss sharing ratios, are an allowable deduction to the partnership and are also separately reported on Schedule K-1 for inclusion on the partner's tax return. Choice "1" is incorrect. Salaries paid to non-partner employees are deducted from revenues to arrive at net ordinary business income or loss at the partnership level. Each partner's distributive share of the net income or loss is then reported on Schedule K-1. Choice "2" is incorrect. Advertising expenditures incurred by the partnership are deducted from revenues to arrive at net ordinary business income or loss at the partnership level. Each partner's distributive share of the net income or loss is then reported on Schedule K-1. Choice "4" is incorrect. Depreciation of assets used in the business is deducted from revenues to arrive at net ordinary business income or loss at the partnership level. Each partner's distributive share of the net income or loss is then reported on Schedule K-1.

Ben Flood, attorney at law, is a sole proprietor and files Schedule C with his federal Form 1040. Which of the following is not a deductible expense on Schedule C? (1) $30 business tax payable to the city in which he practices (2) Salaries paid to the paralegal who works for him (3) Health insurance for him and his family (4) Depreciation on the computer used by his assistant

Choice "3" is correct. A rule of thumb is that personal expenses are not allowed as deductions on the Schedule C. For instance, personal use of an automobile is considered a personal expense, not a deductible expense on Schedule C. Schedule C items should be only those related to the operation of the business itself. Health insurance for himself and his family is actually an adjustment to arrive at adjusted gross income. Choice "1" is incorrect. Business tax items are deductible expenses which should be reported on Schedule C. Choice "2" is incorrect. Salaries and commissions paid to others as part of the business are expenses allowed on Schedule C. Choice "4" is incorrect. Depreciation on business assets is an allowable deduction.

Which of the following conditions must be present in a divorce agreement executed on or before Dec 31, 2018 for a payment to qualify as deductible alimony? I. Payments must be in cash or its equivalents II. The payments must end at the recipient's death. (1) I only (2) II only (3) Both I and II (4) Neither I nor II

Choice "3" is correct. Among the requirements for payments to be classified as alimony are the following: 1. Payment must be in cash or its equivalent. 2. Payments cannot extend beyond the death of the payee-spouse. 3. Payments must be legally required pursuant to a written divorce (or separation) agreement. 4. Payments cannot be made to members of the same household. 5. Payments must not be designated as anything other than alimony. The spouses may not file a joint tax return. Note: The requirements for payments to be considered alimony (income) are the same as for payments to be alimony (deductions). Under the Tax Cut and Jobs Act of 2017, alimony paid is not deductible and alimony received is not considered taxable income for all divorce or separation agreements executed after December 31, 2018.

Bob is a farmer and is required to use the accrual method. At the beginning of the year, Bob has inventory, including livestock held for resale, amounting to $10,000. During the year, Bob purchased livestock totaling $3,000. Bob's ending inventory was $4,000. Bob's net sales for the year totaled $17,000. What is Bob's gross profit for the current year? (1) $9,000 (2) $17,000 (3) $8,000 (4) $13,000

Choice "3" is correct. Bob's beginning inventory 10,000 + purchases 3,000 - 4,000 ending inventory = 9,000 cost of sales. If net sales totaled $17,000, then $17,000 sales - $9,000 cost of sales = $8,000 gross profit. Choice "1" is incorrect. This is equal to Bob's cost of sales based on the change in inventory. Choice "2" is incorrect. This is the amount of Bob's net sales. Gross profit is calculated as sales minus cost of sales. Choice "4" is incorrect. This amount is equal to Bob's net sales less the amount of the ending inventory rather than the change inventory or cost of sales.

Robert Corp. granted an incentive stock option for 200 shares to Beverly, an employee, on March 14, Year 12. The option price and FMV on the date of grant was $150. Beverly exercised the option on August 2, Year 14, when the FMV was $180 per share. She sold the stock on September 20, Year 15, for $250 per share. How much gross income did Beverly recognize in Year 12? (1) $30,000 (2) $150 (3) $0 (4) $20,000

Choice "3" is correct. Due to the fact that this is a qualified stock option, there is no recognition of income in the year of grant. Choice "1" is incorrect. This is the purchase price of the stock upon exercise of 200 shares at $150 per share. It is not income in the year of grant as per the above explanation. Choice "2" is incorrect. This is simply the option price per share on the date of grant. Choice "4" is incorrect. This is the gain Beverly will recognize upon the sale of the stock. The purchase was 200 shares at $150 per share, or $30,000. The sale was 200 shares at $250 per share, or $50,000. This gain is not recognized until the sale occurs in Year 15.

Randolph is a single individual who always claims the standard deduction. Randolph received the following in the current year: Wages $22,000 Unemployment compensation 6,000 Pension distribution (100% taxable) 4,000 A state tax refund from the previous year 425 What is Randolph's gross income? (1) $22,000 (2) $28,425 (3) $32,000 (4) $32,425

Choice "3" is correct. Each item listed here is included in gross income except for the state tax refund from a prior year. The taxpayer always claims the standard deduction. This means that the state tax was not deducted in the year it was paid. Under the tax benefit rule, the refund of that tax is not taxable. Wages 22,000 Unemployment compensation 6,000 Pension distribution (100% taxable) 4,000 Total 32,000

An individual taxpayer reports the following items for the current year: Ordinary income from partnership A, operating a movie theater in which the taxpayer materially participates $70,000 Net loss from partnership B, operating an equipment rental business in which the taxpayer does not materially participate (9,000) Rental income from building rented to a third party 7,000 Short-term capital gain from sale of stock 4,000 What is the taxpayer's adjusted gross income for the year? (1) $70,00 (2) $72,000 (3) $74,000 (4) $77,000

Choice "3" is correct. Except in the year in which an individual, estate, trust, or closely-held C corporation disposes of an entire interest in a passive activity investment, such taxpayers cannot deduct passive activity expenses and losses against income and gain attributable to non-passive activities. A passive activity is (i) any activity in which such taxpayers do not materially participate and (ii) as a general rule, such taxpayers' rental real estate investments, regardless of the extent of such taxpayers' involvement with the rental real estate operations. A limited exception (the "Mom and Pop Exception") regarding rental real estate activities is available to individuals, but the facts of this question do not provide any information which would entitle the taxpayer to the benefits of this exception. Hence, the taxpayer can deduct, against the profit from the taxpayer's $7,000 passive activity rental income from the building rented to a third party, only $7,000 of the $9,000 net loss from partnership B which is operating an equipment rental business in which the taxpayer does not materially participate. Computation of adjusted gross income for the year: Ordinary income from partnership A, operating a movie theater in which the taxpayer materially participates 70,000 Rental income from building rented to a third party (a passive activity) 7,000 Net loss from partnership B, operating an equipment rental business in which the taxpayer does not materially participate(per the above rule the taxpayer can deduct only $7,000 of the $9,000 passive activity loss) (7,000) Short-term capital gain from sale of stock (fully taxable) 4,000 Adjusted gross income for the year 74,000

Dietz is a passive investor in three activities which have been profitable in previous years. The profit and losses for the current year are as follows: Activity X $(30,000) Activity Y (50,000) Activity Z 20,000 Total $(60,000) What amount of suspended loss should Dietz allocate to Activity X?

Choice "3" is correct. For the current year, there is a suspended net passive activity loss (PAL) of $60,000. The loss is allocated between the two activities with passive losses based on the ratio of each activity's loss to the total losses. The suspended PAL allocated to X is $22,500. Activity X $(30,000) Activity Y (50,000) Total losses $(80,000) Activity X: 30,000 / 80,000 = 37.5% × $60,000 = $22,500 Activity Y: 50,000 / 80,000 = 62.5% × $60,000 = $37,500

Garner is a 25 percent partner in Classic General Partnership. On February 3, Garner's tax basis in Classic was $10,000 when she received a nonliquidating distribution of $5,000 cash. Classic had no unrealized receivables, appreciated inventory, or properties that had been contributed by its partners. Classic reported the following for the same year: U.S. Treasury interest $30,000 Ordinary business income 120,000 What amount of income from Classic should Garner include in her gross income for that year? (1) $7,500 (2) $30,000 (3) $37,500 (4) $42,500

Choice "3" is correct. Garner's 25 percent share of the partnership's $120,000 ordinary business income is $30,000. Her 25 percent share of the partnership's $30,000 U.S. Treasury interest is $7,500. These are both included in Garner's gross income for the year. A nonliquidating cash distribution is a nontaxable return of capital to the extent that the partner has basis in his or her partnership interest. Because Garner's basis in her partnership interest is more than the $5,000 cash distribution, the distribution is nontaxable. Garner's total gross income for the year is $30,000 + $7,500 = $37,500. Choice "1" is incorrect. Garner's gross income also includes her 25 percent share of the partnership's ordinary business income of $30,000. Choice "2" is incorrect. Garner's gross income also includes her 25 percent share of the partnership's U.S. Treasury interest income of $7,500. Choice "4" is incorrect. Garner's gross income includes her 25 percent share of the partnership's ordinary business income and U.S. Treasury interest income, but not the $5,000 nonliquidating cash distribution.

DAC Foundation awarded Kent $75,000 in recognition of lifelong literary achievement. Kent was not required to render future services as a condition to receive the $75,000. What condition(s) must have been met for the award to be excluded from Kent's gross income? I. Kent was selected for the award by DAC without any action on Kent's part II. Pursuant to Kent's designation, DAC paid the amount of the award either to a governmental unit or to a charitable organization (1) I only (2) II only (3) Both I and II (4) Neither I nor II

Choice "3" is correct. Generally, the fair market value of prizes and awards is taxable income. However, an exclusion from income for certain prizes and awards applies when the winner is selected for the award without entering into a contest (i.e., without any action on the individual's part) and then assigns the award directly to a governmental unit or charitable organization. Therefore, conditions "I" and "II" must be met in order for Kent to exclude the award from his gross income. Choice "1" is incorrect. "II" is a necessary condition as well. See explanation above. Choice "2" is incorrect. "I" is a necessary condition as well. See explanation above. Choice "4" is incorrect. "I" and "II" are both necessary conditions. See explanation above.

Jensen reported the following items during the current year: Fair rent value of a condominium owned by Jensen's employer $1,400 Cash found in a desk purchased for $30 at a flea marker 400 Inheritance 11,000 The employer allowed Jensen to use the condominium for free in recognition of outstanding achievement. Based on this information, what is Jensen's gross income for the year? (1) $1,400 (2) $1,770 (3) $1,800 (4) $12,400

Choice "3" is correct. Gross income includes employee achievement awards not in the form of tangible personal property. Gross income also includes treasure troves to the extent of its value in United States currency. Choice "1" is incorrect. Gross income includes treasure troves. Choice "2" is incorrect. Gross income includes treasure troves to the extent of its value in United States currency. Choice "4" is incorrect. Gross income does not include inheritances.

Freeman, a single individual, reported the following income in the current year: Guaranteed payment from services rendered to a partnership $ 50,000 Ordinary income from an S corporation 20,000 What amount of Freeman's income is subject to self-employment tax? (1) $0 (2) $20,000 (3) $50,000 (4) $70,000

Choice "3" is correct. Guaranteed payments are reasonable compensation paid to a partner for services rendered (or use of capital) without regard to his ratio of income. Earned compensation is subject to self-employment tax. Payments not guaranteed are merely another way to distribute partnership profits. The ordinary income reported from an S corporation is taxable income to the individual or their own individual tax return but is not subject to self-employment tax. The ordinary income reported from a partnership may be subject to self-employment tax (if to a general partner).

Guaranteed payment by a partnership to a partner for services rendered may include an agreement to pay: I. A salary of $5,000 monthly without regard to partnership income II. A 25% interest in partnership profits (1) I only (2) II only (3) Both I and II (4) Neither I nor II

Choice "3" is correct. Guaranteed payments to partners are deductible on Form 1065 to arrive at partnership ordinary income. On Schedule K-1, guaranteed payments are shown as income and flow through as ordinary income.

During the current year, Freda was entirely supported by her three children, Michelle, Brian, and John, who provided support in the following percentages: Michelle 10% Brian 48% John 42% Which of the children is entitled to claim their mother as a dependent, assuming a multiple support agreement exists? (1) Michelle (2) Michelle or John (3) Brian or John (4) Michelle, Brian, or John

Choice "3" is correct. In a multiple support agreement, all must be qualifying relatives who together contribute more than 50% of the support of the dependent. In addition, a contributor must have provided more than 10% of the individual's support to claim the individual as a dependent. Choice "1" is incorrect. Michelle must contribute more than 10%. This answer also does not take into account Brian or John, both of whom are qualifying relatives and are able to take the dependency exemption. Choice "2" is incorrect. Michelle is not able to take the exemption and Brian is able to take the exemption. Choice "4" is incorrect. Michelle is not able to take the exemption as she did not contribute more than 10% of Freda's support.

In a tax year where the taxpayer pays qualified education expenses, interest income on the redemption of qualified U.S. Series EE Bonds may be excluded from gross income. The exclusion is subject to a modified gross income limitation and a limit of aggregate bond proceeds in excess of qualified higher-education expenses. Which of the following is (are) true? I. The exclusion applies for education expenses incurred by the taxpayer, the taxpayer's spouse, or any person whom the taxpayer may claim as a dependent for the year. II. "Otherwise qualified higher-education expenses" must be reduced by qualified scholarships not includible in gross income. (1) I only. (2) II only. (3) Both I and II. (4) Neither I nor II.

Choice "3" is correct. Interest earned on Series EE bonds issued after 1989 may qualify for exclusion. One requirement is that the interest is used to pay tuition and fees for the taxpayer, spouse, or dependent enrolled in higher education. The interest exclusion is reduced by qualified scholarships that are exempt from tax and other nontaxable payments received for educational expenses (other than gifts and inheritances).

On March 1 of Year 0, Judy was granted an incentive stock option (ISO) to purchase 50 shares of her employer's stock for $10 per share. The FMV of the stock on the date of the grant was $12 per share. On May 1 of Year 1, Judy exercised her option when the stock was selling for $15 per share. On July 1 of Year 2, Judy sold all of the shares for $20 per share. What amount and character of income does Judy recognize in Year 2? (1) $0 (2) $500, ordinary (3) $500, long-term capital gain (4) $600, ordinary

Choice "3" is correct. Judy will recognize a $500 long-term capital gain in Year 2. Her basis in the stock is $500 (50 shares × $10/share). She does not recognize income in Year 0 when the options are received because the options qualify as incentive stock options. When Judy sells the stock in Year 2, she recognizes a $500 long-term capital gain. This is the $1,000 (50 shares × $20/share) selling price less her adjusted basis of $500. Choice "1" is incorrect. The sale of shares of stock is a taxable event. Choice "2" is incorrect. Because Judy held the shares for more than one year, the gain recognized is a long-term capital gain. Choice "4" is incorrect. Judy will recognize a $500 long-term capital gain in Year 2. Her basis in the stock is $500 (50 shares × $10/share). When Judy sells the stock in Year 2, she recognizes a $500 long-term capital gain. This is the $1,000 (50 shares × $20/share) selling price less her adjusted basis of $500.

Evan, an individual, has a 40% interest in EF, an S corporation. At the beginning of the year, Evan's basis in EF was $2,000. During the year, EF distributed $100,000 and reported operating income of $200,000. What amount should Evan include in gross income? (1) $38,000 (2) $40,000 (3) $80,000 (4) $118,000

Choice "3" is correct. Like partnerships, S corporations report both separately and non-separately stated items of income and/or loss. Allocations to shareholders are made on a per-share, per-day basis in accordance with ownership percentage. Shareholders in an S corporation must include on their personal income tax return their distributive share of each separate "pass-through" item. Shareholders are taxed on these items, regardless of whether or not these items have been distributed to them during the year. EF's operating income 200,000 x Evan's ownership % 40% Gross income for Evan 80,000 Choice "1" is incorrect. This answer option incorrectly assumes that Evan's gross income is calculated as 40% of the distribution for the year ($100,000 x 40% = $40,000) less the basis of $2,000 as of the beginning of the year ($40,000 - $2,000 = $38,000). Choice "2" is incorrect. This answer option incorrectly assumes that Evan's gross income is calculated as 40% of the distribution for the year ($100,000 x 40% = $40,000). Choice "4" is incorrect. This answer option incorrectly assumes that Evan's gross income is 1 - 40%, or 60%, of the corporation's operating income for the year ($200,000 x 60% = $120,000), less the basis of $2,000 as of the beginning of the year ($120,000 - $2,000 = $118,000).

Nicole and Andrew Harris contribute to more than half of the support of their three children, Travis, Luke, and John. Travis, age 20, worked full time at the local deli and earned $20,000. Luke, 18, is a part-time college student who earned $5,000 working as a resident assistant in the student dormitory where he lived half of the year. John, age 25, is an aspiring actor who lives at home with Nicole and Andrew. John earned $2,500 for the three commercials he starred in. Who qualifies as a dependent for Nicole and Andrew under either the rules of qualifying child or qualifying relative? (1) Travis (2) Travis and Luke (3) Luke and John (4) Travis, Luke, and John

Choice "3" is correct. Luke and John satisfy dependency requirements: Travis does not meet the age limit for qualifying child. His income is over the gross income limitation for qualifying relative. Luke meets the qualifying child rules (CARES). He is under the age of 19 and only lives away from home while at college. John meets the qualifying relative rules (SUPORT). His parents provide more than half of his support and his gross income is under the limitation.

In year 4 , after Mindy's three children have grown and moved out of the house, Mindy (unmarried) moved her mother, Mary, into an assisted living facility for which Mindy pays 75% of the cost. Mindy had not previously lived with Mary, and Mary paid for her own living expenses while she lived in her own home. What filing status should Mindy use for Year 4, assuming Mary moved into the assisted living facility on January 1, Year 4? (1) Single (2) Married filing jointly (3) Head of household (4) Surviving spouse

Choice "3" is correct. Mindy qualifies for and should use head of household status in Year 4, because she maintained more than half of the upkeep on Mary's principal residence for the entire taxable year (note that Mindy is not required to live with her mother to qualify for head of household status). It is the most favorable filing status for which she qualifies. Choice "1" is incorrect. Mindy qualifies for a more favorable filing status than single. Choice "2" is incorrect. Mindy is not married. Choice "4" is incorrect. Mindy has not had a spouse die in the past two years.

Mort and Mindy met at a New Year's Eve party held December 31, Year 1. They instantly bonded, fell madly in love, and were married at 11:38 p.m that night. Sadly, Mort passed away November 15, Year 2. In January, Year 3, Mindy gave birth to triplets Mark, Mandy, and Maureen. Assuming that Mindy has not remarried, what filing status should she use for Year 5? (1) Single (2) Married filing jointly (3) Head of household (4) Qualifying widow(er) with dependent child

Choice "3" is correct. Mindy should file using the head of household status. She has dependent children living with her, and no longer qualifies as married or as a surviving spouse. Head of household is the most favorable filing status for which she qualifies. Choice "1" is incorrect. Mindy qualifies for a more favorable filing status than single. Choice "2" is incorrect. Mindy is no longer married and Mort did not die in Year 5, so she is not eligible for the married filing jointly status. Choice "4" is incorrect. At Year 5, more than two years have passed since Mort's death so Mindy no longer qualifies for surviving spouse (qualifying widow) status.

What are the treatment options for a net operating loss occurring in tax years after December 31, 2017? (1) Two-year carryback and 20-year carryforward (2) No carryback and 20-year carryforward (3) No carryback and indefinite carryforward (4) Two-year carryback and indefinite carryforward

Choice "3" is correct. Net operating losses may not be carried back (for tax years beginning after December 31, 2017), but can be carried forward indefinitely. Also, note that the net operating loss utilized in one tax year is limited to 80 percent of taxable income.

A review of Bearing's Year 2 records disclosed the following tax information: Wages $18,000 Taxable interest and qualifying dividends 4,000 Schedule C trucking business net income 32,000 Rental (loss) from residential property (35,000) Limited partnership (loss) (5,000) Bearing actively participated in the rental property and was a limited partner in the partnership. Bearing had sufficient amounts at risk for the rental property and the partnership. What is Bearing's Year 2 adjusted gross income? (1) $14,000 (2) $19,000 (3) $29,000 (4) $54,000

Choice "3" is correct. Passive activity losses (PALs) can only be deducted up to passive activity income. There is no passive activity income indicated. Therefore, the passive loss from the partnership is not deductible. $25,000 of the $35,000 rental real estate loss is deductible under the "mom and pop" exception because Bearing actively participates in the rental property and the AGI is below the phaseout amounts. The AGI is calculated as follows: Wages 18,000 Taxable interest and qualifying dividends 4,000 Schedule C trucking business net income 32,000 Rental loss from residential property (25,000) Adjusted gross income (AGI) 29,000

The Griffins own a mountain cabin that is used for both personal and rental purposes. In the current year, the Griffins rented the cabin out for 150 days and used it personally for 50 days. Assume that the Griffins itemize their deductions. Which of the following statements regarding the treatment of the mountain cabin on the Griffin's tax return is true? (1) 100% of the utilities for the mountain cabin for the entire year are deductible (2) Depreciation is deductible under all rental circumstances (3) Real estate taxes are deductible under all rental circumstances (4) The rental income received is not included in gross income

Choice "3" is correct. Real estate taxes and mortgage interest are either deducted on the rental schedule E or as an itemized deduction (subject to limitations). Therefore, real estate taxes will be deductible whether the Griffins rent their cabin or not. Choice "1" is incorrect. Only the rental portion of the utilities is deducted on Schedule E. Because the Griffins rented their cabin for more than 15 days and used the cabin for the greater of (1) 14 days or (2) more than 10% of the rental days, their cabin is treated as a personal/rental residence. The rental portion is deducted on Schedule E. The personal use portion of the utilities expense is not deductible. Choice "2" is incorrect. Depreciation is only deductible as a rental expense on Schedule E. If the Griffins had rented their cabin out for fewer than 15 days, then the cabin would be treated as a personal residence. As a personal residence, depreciation is not deductible. Therefore, depreciation is not deductible under all rental circumstances. Choice "4" is incorrect. Because the Griffins rented their cabin for more than 15 days and used the cabin for the greater of (1) 14 days or (2) more than 10% of the rental days, their cabin is treated as a personal/rental residence. The Griffins would include the rental income received in their gross income on Schedule E.

Carson owned 40% of the outstanding stock of a C corporation. During a tax year, the corporation reported $400,000 in taxable income and distributed a total of $70,000 in cash dividends to its shareholders. Carson accurately reported $28,000 in gross income on Carson's individual tax return. If the corporation had been an S corporation and the distributions to the owners had been proportionate, how much income would Carson have reported on Carson's individual return? (1) $28,000 (2) $132,000 (3) $160,000 (4) $188,000

Choice "3" is correct. S Corporations work in a similar fashion to partnerships. The income is passed through to the shareholder and included in taxable income whether or not it is actually distributed. Therefore, Carson will report 40% of the $400,000 taxable income, or $160,000. The $28,000 distribution will not affect the taxable income, but will reduce Carson's basis in the S Corporation stock. Choice "1" is incorrect. This is simply the same answer as if the corporation were a C Corporation. Choice "2" is incorrect. This is the correct answer of $160,000 reduced by the $28,000 distribution. The $28,000 will not reduce taxable income, but will reduce Carson's basis in the S Corporation stock. Choice "4" is incorrect. This is the correct answer of $160,000 increased by the $28,000 distribution. The $28,000 will not increase taxable income, but will reduce Carson's basis in the S Corporation stock.

Which of the following types of expenditures is not subject to uniform capitalization (UNICAP) rules? (1) Compensation for manufacturing labor (2) Cost of direct materials (3) Administrative expenditures (4) Compensation for production supervisors

Choice "3" is correct. Selling, advertising, marketing, general, administrative, and research and development costs are not subject to UNICAP. Direct materials, direct labor, and factory overhead costs are capitalized as inventory under the UNICAP rules. Choice "1" is incorrect. Compensation for manufacturing labor is a direct labor cost and is required to be capitalized under UNICAP. Choice "2" is incorrect. The cost of direct materials must be capitalized under UNICAP. Choice "4" is incorrect. Compensation for production supervisors is a factory overhead cost and therefore must be capitalized under UNICAP.

Seth Silver had the following items of income during the taxable year: Interest income from a checking account $1,000 Interest income from a money market account 2,050 Interest income from a municipal bond he purchased during the current year 250 Interest income from federal bonds he purchased two years ago 750 On his current year tax return, what amount is taxable income? (1) $3,050 (2) $3,300 (3) $3,800 (4) $4,050

Choice "3" is correct. Taxable interest includes amounts received from general investment accounts as well as interest on federal obligations. Interest received from state and municipal bonds is not taxable.

Dale received $1,000 in the current year for jury duty. In exchange for regular compensation from her employer during the period of jury service. Dale was required to remit the entire $1,000 to her employer in this year. In Dale's current year income tax return, the $1,000 jury duty fee should be: (1) Claimed in full as na itemized deduction (2) Excluded from Dale's tax return (3) Deducted from gross income in arriving at adjusted gross income (4) Included in taxable income without a corresponding offset against other income

Choice "3" is correct. The $1,000 jury duty fee that was required to be remitted to the employer may be deducted from gross income in arriving at adjusted gross income. This, in effect, washes out the $1,000 income she will have to report as part of gross income for the jury duty fees paid to her. Choice "1" is incorrect. The amount remitted is allowed as an adjustment in arriving at AGI, not as an itemized deduction. Choice "2" is incorrect. The amount remitted is allowed as an adjustment in arriving at AGI. Choice "4" is incorrect. A corresponding offset is allowed against other income as an adjustment in arriving at AGI.

Perle, a dentist, billed Wood $600 for dental services. Wood paid Perle $200 cash and built a bookcase for Perle's office in full settlement of the bill. Wood sells comparable bookcases for $350. What amount should Perle include in taxable income as a result of this transaction? (1) $0 (2) $200 (3) $550 (4) $600

Choice "3" is correct. The $200 cash received plus the $350 fair value of the bookcase received must be included in income by Perle, for a total of $550. The income is based on the value in money or fair value of property received by Perle, not the $600 billed. Choice "1" is incorrect. Perle must report taxable income as a result of this transaction. Choice "2" is incorrect. The $350 fair value of the bookcase received is also income for Perle. Choice "4" is incorrect. The income is based on the total value received by Perle, not the $600 billed.

Jasmin purchased 100 shares of Pinkstey Corporation (publicly traded company) on January 1 of Year 1 for $5,000. The FMV of the shares at the end of Year 1 was $6,000. On January 1 of year 4, Pinkstey Corporation declared a 2-for-1 stock split when the fair market value of the stock was $65 per share. On January 1 of Year 5, Jasmin sold all of her Pinkstey Corporation stock when the fair market value was $40 per share. Which of the following statements is true? (1) Jasmin reports $6,500 in gross income for the 2-for-1 stock split in Year 4 (2) Jasmin's basis in the Pinkstey Corporation stock at the end of Year 4 is $65/share (3) Jasmin has no taxable income for the Pinkstey Corporation stock in Year 4 (4) Jasmin owns 100 shares in Pinkstey Corporation stock at the end of year 4. Jasmin has no taxable income for the Pinkstey Corporation stock in Year 4

Choice "3" is correct. The 2-for-1 stock split is not a taxable event. After the split, Jasmin has 200 shares of Pinkstey Corporation stock with a basis of $25/share. Jasmin is taxed in Year 5 on the sale of the stock: $40 x 200 shares = $8000 - $5000 stock basis (200 shares x $25/share) = $3000 long-term capital gain. Choice "1" is incorrect. The 2-for-1 stock split is not a taxable event. Choice "2" is incorrect. After the Year 4 stock split, Jasmin's basis in the stock is $25/share ($50 original basis/2). She owns 200 shares after the split. Choice "4" is incorrect. After the Year 4 stock split, Jasmin owns 200 shares with a $25/share basis ($50 original basis/2).

Which of the following types of costs are required to be capitalized under the Uniform Capitalization Rules of Code Sec. 263A? (1) Marketing (2) Distribution (3) Warehousing (4) Office maintenance

Choice "3" is correct. The Uniform Capitalization Rules require the capitalization of certain costs related to inventory. They include direct materials, direct labor, and indirect overhead costs including warehousing. Choice "1" is incorrect. Marketing is a selling expense, which is not required to be capitalized under the Uniform Capitalization Rules. Choice "2" is incorrect. Distribution is a selling expense, which is not required to be capitalized under the Uniform Capitalization Rules. Choice "4" is incorrect. Office maintenance is a general and administrative expense, which is not required to be capitalized under the Uniform Capitalization Rules.

With regard to the inclusion of Social Security benefits in gross income, for the Year 8 tax year, which of the following statements is correct? (1) The Social Security benefits in excess of modified adjusted gross income are included in gross income (2) The Social Security benefits in excess of one half the modified adjusted gross income are included in gross income (3) Eighty-five percent of the Social Security benefits is the maximum amount of benefits to be included in gross income (4) The Social Security benefits in excess of the modified adjusted gross income over a threshold amount are included in gross income

Choice "3" is correct. The amount of Social Security benefits that is taxed is dependent on whether the combined income (AGI plus interest on tax-exempt bonds and 50 percent of the Social Security benefits) is greater than a threshold amount. If the combined income is less than the threshold, the amount taxed is the lesser of 1) 50 percent of the benefits; or 2) 50 percent of the excess of the combined income over the threshold. If the combined income is greater than the threshold, the amount taxed is the lesser of 1) amount calculated above plus 85 percent of the excess of the combined income over the threshold; or 2) 85 percent of the benefits. Thus, 85 percent of the benefits is the maximum amount of benefits that may be included in gross income.

What is the basic deduction calculation for the qualifying business income deduction? (1) 30% x Qualifying business income (QBI) (2) 20% x W-2 wages (3) 20% x Qualifying business income (QBI) (4) 30% x W-2 wages

Choice "3" is correct. The basic calculation for the QBI deduction is 20% × QBI. The deduction is subject to limitations.

Mosh, a sole proprietor, uses the cash method of accounting. At the beginning of the current year, accounts receivable were $25,000. During the year, Mosh collected $100,000 from customers. At the end of the year, accounts receivable were $15,000. What was Mosh's gross taxable income for the current year? (1) $75,000 (2) $90,000 (3) $100,000 (4) $110,000

Choice "3" is correct. The facts state that cash collections from customers were $100,000 and as a cash basis taxpayer this is the amount of Mosh's gross taxable income for the year. Note that according to the formula BASE - we can determine the amount of sales = $90,000, but that would give us accrual, not cash basis, income. Beginning A/R 25,000 Add―Sales 90,000 accrual basis taxable income 115,000 Subtract―Cash collections (100,000) cash basis taxable income Ending A/R 15,000 Choices "1" and "4" are incorrect. See explanation above. Choice "2" is incorrect. $90,000 is the amount of sales that would be Mosh's taxable income if Mosh were an accrual basis taxpayer.

Johnson worked for ABC Co. and earned a salary of $100,000. Johnson also received, as a fringe benefit, group term-life insurance at twice Johnson's salary. Assume the annual IRS-established uniform cost of insurance is $2.76 per $1,000. What amount must Johnson include in gross income? (1) $100,000 (2) $100,276 (3) $100,414 (4) $100,552

Choice "3" is correct. The first $50,000 of group term life insurance is a nontaxable fringe benefit. Amounts exceeding this are taxable based on IRS tables. The total group term life insurance here is $200,000 (twice the salary of $100,000). The amount exceeding $50,000 is $150,000. The cost given here is $2.76 per $1,000 of insurance. 150 × $2.76 = $414. So the total amount included in gross income is $100,414 ($100,000 + $414). Choice "1" is incorrect. $100,000 does not include any of the taxable amount of group term life insurance. Choice "2" is incorrect. $100,276 only includes $100,000 of the group term life insurance instead of $150,000. Choice "4" is incorrect. $100,552 includes the entire $200,000 of the group term life insurance instead of only $150,000.

Which represents the correct order of deductions in the individual income tax formula? (1) Standard deduction or Itemized deductions, Adjustments, QBI deduction (2) QBI deduction, Adjustments, Standard deduction or Itemized deductions (3) Adjustments, Standard deduction or Itemized deductions, QBI deduction (4)Adjustments, Standard deduction, Itemized deductions, QBI deduction

Choice "3" is correct. The individual income tax formula is: Gross income - Adjustments = Adjusted gross income - Standard deduction or Itemized deductions = Taxable income before QBI Deduction - QBI deduction = Taxable Income Choice "1" is incorrect. Adjustments are applied first to arrive at AGI. Then, either the standard deduction or itemized deductions are subtracted from AGI. The QBI deduction is applied last to arrive at taxable income. Choice "2" is incorrect. Adjustments are applied first to arrive at AGI. Then, either the standard deduction or itemized deductions are subtracted from AGI. The QBI deduction is applied last to arrive at taxable income. Choice "4" is incorrect. Either the standard deduction or itemized deductions can be subtracted from Adjusted gross income, not both.

Flowers, a married taxpayer, purchased an annuity for $64,400 that will pay $700 per month over the life of Flowers and Flowers's spouse. At the time of purchase, the couple's joint life expectancy was 23 years. Flowers received payment beginning April 1, Year 1, amounting to $6,300 in the first year of the annuity contract. How much is includible in Flowers's gross income in the first year? (1) $0 (2) $2,100 (3) $4,200 (4) $6,300

Choice "3" is correct. The investment amount is divided over the number of months of expected recovery. $64,400 / 276 = $233.33 (23 years × 12 months = 276 months). This is the amount of each payment that is a cost recovery and not taxable. Payments began in April of this year, resulting in 9 payments this year ($233.33 × 9 = $2,100). This is the portion of the $6,300 that is not taxable. So the taxable amount is $4,200 ($6,300 - $2,100 = $4,200). Choice "1" is incorrect. Zero indicates that all of the payments are not taxable. But only the prorated amount of the cost recovery is not taxable. Choice "2" is incorrect. $2,100 is the nontaxable portion of the payment based on the above computation. Choice "4" is incorrect. $6,300 is the total amount received. But the prorated portion of the cost recovery is not taxable.

The rule limiting the allowability of passive activity losses and credits applies to: (1) Partnerships (2) S corporations (3) Personal service corporations (4) Widely-held C corporations.

Choice "3" is correct. The rule limiting the allowability of passive activity losses and credits applies to personal service corporations. Choice "1" is incorrect. The passive activity limitations apply to the various partners in the partnership as opposed to the partnership itself. Choice "2" is incorrect. The passive activity limitations apply to the various shareholders in the S corporation as opposed to the corporation itself. Choice "4" is incorrect. The passive activity rules do not apply to widely held C corporations.

Logan, an employee of Argon Industries, earned a salary of $60,000 in Year 2. In addition, the following two transactions between Logan and Argon occurred in Year 2: Logan received a bonus of 100 shares of publicly traded stock worth $13,000 with a basis to Argon of $8,000, and Logan purchased 1,000 shares of unrestricted Argon stock pursuant to a nonqualifying stock option plan for $10 per share when the stock was valued at $25 per share. What amount of compensation should Argon report in Logan's Form W-2 for Year 2? (1) $60,000 (2) $73,000 (3) $88,000 (4) $93,000

Choice "3" is correct. The salary of $60,000 is included in the Form W-2. The FMV of the bonus of $13,000 is included in the Form W-2. Because the stock option was nonqualifying, the bargain element is included in Form W-2 as well. The stock is worth $25 per share and the option price is $10 per share. That is a bargain element on nonqualified stock options of $15 per share on 1,000 shares. That is $15,000. $60,000 + $13,000 + $15,000 = $88,000. Choice "1" is incorrect. $60,000 ignores the bonus and the stock option. Choice "2" is incorrect. $73,000 ignores the stock option. Choice "4" is incorrect. $93,000 is not correct based on the above rule.

During the current year, Adler had the following cash receipts: Wages $ 18,000 Interest income from investments in municipal bonds 400 Unemployment compensation 3,900 What is the total amount that must be included in gross income on Adler's current year income tax return? (1) $18,000 (2) $18,400 (3) $21,900 (4) $22, 300

Choice "3" is correct. The wages of $18,000 and unemployment compensation of $3,900 are both includable in gross income on Adler's current year income tax return. Choice "1" is incorrect. The unemployment compensation must be included in gross income. Choice "2" is incorrect. Municipal bond interest income is excluded from gross income, and the unemployment compensation must be included in gross income. Choice "4" is incorrect. Municipal bond interest income is excluded from gross income.

Which of the following types of property is not subject to the uniform capitalization (UNICAP) rules beginning in 2018? (1) Manufacturer's inventory (2) Retailer's inventory with annual average gross receipts of $30,000,000 (past three years) (3) Retailer's inventory with annual average gross receipts of $5,000,000 (past three years) (4) Tangible personal property produced by a taxpayer for use in the taxpayer's trade or business

Choice "3" is correct. UNICAP rules do not apply to a retailer's inventory if the retailer's average gross receipts for the preceding three tax years do not exceed $26,000,000 annually. Choice "1" is incorrect. Real or tangible personal property produced by the taxpayer for sale to his or her customers (manufacturer's inventory) are subject to the UNICAP rules. Choice "2" is incorrect. Real or tangible person property acquired by the taxpayer for resale (retailer's inventory) are all subject to the UNICAP rules. However, retailers with average gross receipts under $26,000,000 for the three preceding tax years are exempt. Choice "4" is incorrect. Real or tangible person property produced by the taxpayer for use in his or her trade or business is subject to the UNICAP rules.

Don Wolf became a general partner in Gata Associates on January 1, 2017, with a 5% interest in Gata's profits, losses, and capital. Gata is a distributor of auto parts. Wolf does not materially participate in the partnership business. For the year ended December 31, 2017, Gata had an operating loss of $100,000. In addition, Gata earned interest of $20,000 on a temporary investment. Gata has kept the principal temporarily invested while awaiting delivery of equipment that is presently on order. The principal will be used to pay for this equipment.Wolf's passive loss for 2017 is: (1) $0 (2) $4,000 (3) $5,000 (4) $6,000

Choice "3" is correct. Wolf's passive loss for the current year is $5,000 ($100,000 operating loss × 5% interest in partnership). Choice "1" is incorrect. Wolf did not materially participate in the partnership, so the loss was passive. Choice "2" is incorrect. Wolf's passive loss of $5,000 could not be reduced by his distributive share of the partnership's "interest income" totaling $1,000. Interest income is considered "portfolio income," and neither the partnership nor a partner can offset it against passive losses. Choice "4" is incorrect. No items of income or deduction from portfolio income or activities in which the taxpayer materially participates may be combined or offset with passive losses unless the activity generating the loss is completely disposed of in a taxable transaction.

Calculate the taxpayer's qualifying business income (QBI) deduction for a qualified trade or business (QTB): Filing status: Single Taxable income before QBI deduction: $185,800 Net capital gains: $0 Qualified business income (QBI): $80,000 QTB's W-2 wages: $20,000 (1) $16,000 (2) $10,000 (3) $2,700 (4) $13,300

Choice "4" is correct. Taxable income before the QBI deduction is within the phase-in range of $163,300 to $213,300 (single) for 2020, so the QBI deduction is reduced. Tentative QBI deduction: $80,000 (QBI) × 20% = $16,000 W-2 wage limitation: $20,000 × 50% = $10,000 $16,000 − $10,000 = $6,000 excess amount Calculation of phase-in percentage: $185,800 taxable income − $163,300 = $22,500 $22,500 / $50,000 (phase-in range) = 45% $6,000 excess amount × 45% phase-in percentage = $2,700 reduction amount $16,000 tentative QBI deduction − $2,700 reduction amount = $13,300 reduced QBI deduction

Calculate the taxpayer's qualifying business income deduction for a qualified trade or business: Filing status: Single Taxable income: $100,000 Net capital gains: $0 Qualified business income (QBI): $30,000 W-2 wages: $10,000 (1) $5,000 (2) $70,000 (3) $20,000 (4) $6,000

Choice "4" is correct. $30,000 QBI × 20% = $6,000. W-2 wage and property limits do not apply to qualified trade or businesses with income below the taxable income threshold (2020 threshold for single taxpayers = $163,300). Choice "1" is incorrect. The W-2 wage and property limitations do not apply to a qualified trade or business under the taxable income limitations. Therefore, the deduction for QBI is not limited to $5,000 (W-2 wages of $10,000 × 50% = $5,000). Choice "2" is incorrect. $30,000 QBI × 20% = $6,000 Choice "3" is incorrect. $30,000 QBI × 20% = $6,000

Which of the following statements is true regarding taxpayers with taxable income below the taxable income limitations for the qualifying business income (QBI) deduction? (1) QBI deduction is only allowed if a qualified trade or business (QTB) (2) QBI deduction is a phased-out deduction if a specified service trade or business (SSTB) (3) QBI deduction is limited to 50 percent of W-2 wages (4) A qualified trade or business (QTB) and specified trade or business (SSTB) are treated the same

Choice "4" is correct. A QTB and an SSTB are treated the same for taxpayers under the taxable income thresholds for the QBI deduction. Choice "1" is incorrect. For taxpayers under the taxable income limitations, the QBI deduction is allowed if the taxpayer is a QTB or an SSTB. Choice "2" is incorrect. The QBI deduction is not phased out for an SSTB if the taxpayer is under the taxable income limitations. Choice "3" is incorrect. The QBI deduction is not limited to 50 percent of wages if the taxpayer is under the taxable income limitations.

Parker, whose spouse died during the preceding year, has not remarried. Parker maintains a home for a dependent child. What is Parker's most advantageous filing status? (1) Single (2) Head of household (3) Married filing separately (4) Qualifying widow(er) with dependent child

Choice "4" is correct. A qualifying widow(er) is a taxpayer who may use the joint tax return standard deduction and rates for each of two taxable years following the year of death of his or her spouse, unless he or she remarries. The surviving spouse must maintain a household that, for the entire taxable year, was the principal place of abode of a son, stepson, daughter, or stepdaughter (whether by blood or adoption). The child must be considered either a qualifying child or a qualifying relative. Parker may file as a qualifying widow(er) because her spouse died in the previous tax year, she did not remarry, and she maintained a home for a dependent child. Because qualifying widow(er) is the most advantageous status and Parker qualifies, Parker would file as a qualifying widow(er). Choice "1" is incorrect. Even though Parker would qualify as single, filing single would give Parker a higher tax liability than the qualifying widow(er) status and therefore is not most advantageous. Choice "2" is incorrect. Parker would not qualify as head of household for the first two years after the death of Parker's spouse because one of the requirements for head of household status is that the taxpayer is not a surviving spouse. (Also, note that the likely reason for this requirement is that filing as head of household status would give the qualifying surviving spouse taxpayer a higher tax liability than the qualifying widow(er) status, which would be less advantageous.) Choice "3" is incorrect. Parker would not qualify to file married filing separately.

Calculate the taxpayer's qualifying business income deduction for a specified service trade or business: Filling status: Single Taxable income: $300,000 Net capital gains: $0 Qualified business income (QBI): $50,000 W-2 wages: $10,000 (1) $15,000 (2) $5,000 (3) $60,000 (4) $0

Choice "4" is correct. A specified service trade or business with taxable income over the $213,300 for 2020 is not eligible for the QBI deduction. Choice "1" is incorrect. A specified service trade or business with taxable income over the upper threshold $213,300 for 2020 is not eligible for the QBI deduction. Choice "2" is incorrect. A specified service trade or business with taxable income over the upper threshold $213,300 for 2020 is not eligible for the QBI deduction. Choice "3" is incorrect. A specified service trade or business with taxable income over the upper threshold $213,300 for 2020 is not eligible for the QBI deduction.

Hunter has a loss of $50,000 from his landscaping business in the current year. He reports the loss on Schedule C of his Form 1040. After deducting the loss against his other sources of income, he has a remaining business loss of $10,000. What are Hunter's options regarding the remaining $10,000 business loss? (1) He can carry the loss back four years and forward 20 years (2) He can carry the loss back two years and forward 20 years (3) He can choose to only carry the loss forward 20 years (4) He can carry the loss forward indefinitely

Choice "4" is correct. A taxpayer's deductible losses are limited each year to $250,000 (unmarried) and $500,000 (married). Hunter's $50,000 loss is under this limitation. The $10,000 remaining business loss may be carried forward indefinitely and is limited to offsetting up to 80 percent of taxable income each subsequent year. Choice "1" is incorrect. There is no four-year carryback for net operating losses. Choices "2" and "3" are incorrect. A taxpayer may carry a net business loss forward indefinitely and may offset up to 80 percent of taxable income in the subsequent years.

Darr, an employee of Source C corporation, is not a shareholder. Which of the following should be included in a taxpayer's gross income? (1) Employer-provided medical insurance coverage under a health plan (2) A $10,000 gift from the taxpayer's grandparents (3) The fair market value of land that the taxpayer inherited from an uncle (4) The dividend income on shares of stock that the taxpayer received for services rendered

Choice "4" is correct. An individual receiving common stock for services rendered must recognize the fair market value as ordinary income. Any dividends received on that stock would also result in income recognition. Choice "1" is incorrect. Employer-provided medical insurance is a tax-free fringe benefit. Choices "2" and "3" are incorrect. Gifts and inheritances are both tax-free to the recipient. (Remember, tax is often paid by the person giving the gift or the estate at death.)

Dave and Pam Stevens contributed to the support of their three children, Lisa, Tanya, and Hannah, and Pam's divorced mother, Ellen. For the current year, Lisa, a 26-year-old sales clerk, earned $27,000. Tanya, 2 23-year-old, full-time college graduate student in accounting, earned $35,000 working for a CPA firm. Hannah, a 20-year-old artist, earned nothing during the year, but is still aspiring to sell her first piece and has signed on with an art studio. Ellen received $10,000 in nontaxable social security benefits and $2,000 in dividend income. All are U.S. citizens and are over half supported by Dave and Pam. How many dependents do Dave and Pam Stevens have under the qualifying child and qualifying relative rules? (1) Zero (2) One (3) Two (4) Three

Choice "4" is correct. Based on the CARES (QC) and the SUPORT (QR) tests, Dave and Pam have three dependents. Lisa: NO. Lisa fails the age limit for QC and exceeds the gross income limitation for QR. Tanya: YES. Tanya meets all tests of QC. She is a full-time student under the age of 24 so she meets the age test. Hannah: YES. Hannah meets all criteria for QR. She fails the age limit test for QC. Ellen: YES. Ellen meets the gross income limitation for QR because the Social Security income is nontaxable and not included for the gross income test. Tanya, Hannah, and Ellen all meet dependency requirements.

Baum, an unmarried optometrist and sole proprietor of Optics, buys and maintains a supply of eyeglasses and frames to sell in the ordinary course of business. In the current year, Optics had $350,000 in gross business receipts and its year-end inventory was not subject to the uniform capitalization rules. Baum's current year adjusted gross income was $90,000 and Baum qualified to itemize deductions. During the year, Baum recorded the following information: Business expenses: Optics cost of goods sold $35,000 Optics rent expense 28,000 Liability insurance premium on Optics 5,250 Other expenditures: Baum's self-employment tax $29,750 Baum's self-employment health insurance 8,750 Insurance premium on personal residence. In the current year, Baum's home was totally destroyed by fire. The furniture had an adjustment basis of $14,000 and a fair market value of $11,000. During the year, Baum collected $3,000 in insurance reimbursement and had no casualty gains during the year. 2,625 Qualified mortgage interest on a loan secured in 2016 to acquire a personal residence 52,500 Annual interest on a $70,000, 5-year home equity loan by a substantial amount. The proceeds were used to purchase a car for personal use 3,500 Points prepaid on January 2 of the current year to acquire the home equity loan 1,400 Real estate taxes on personal residence Estimated payments of current year federal income taxes 13,500 Local property taxes on the car value, used exclusively for personal use What about should Baum report as current year net earnings from self-employment? (1) $243,250 (2) $252,000 (3) $273,000 (4) $281,750

Choice "4" is correct. Baum should report $281,750 as current year net earnings from self-employment (line 12 of the Form 1040), calculated as follows: Gross business receipts 350,000 Cost of goods sold (35,000) Rent expense (28,000) Liability insurance premium (5,250) Net earnings on Schedule C 281,750 Choices "1", "2", and "3" are incorrect. Self-employment tax and self-employment health insurance expenses are adjustments from total gross income. They are not deducted from self-employment earnings. Note: There are many distracters in this question, all relating to items that are either deductible as part of itemized deductions or not deductible. Be careful to read the requirement of the question before spending unnecessary time on the question. The statement that Baum's year-end inventory was not subject to the uniform capitalization rules is a distracter as well. There is not enough information given in the facts to apply the rules if he had been subject to them.

Four years ago, when Cox's spouse died, Cox filled a joint tax return for that year. Cox did not remarry, but continued to provide fill support for a minor child who has been living with Cox. What is Cox's most advantages filing status for the current year? (1) Single (2) Married filing separately (3) Surviving spouse (4) Head of household

Choice "4" is correct. Because Cox is not married, Cox cannot file as married filing separately (or married filing jointly). Likewise, since Cox's spouse died more than two years ago, Cox cannot file as a surviving spouse. Cox may file as single. In addition, since Cox is not married, is not a surviving spouse, and maintains a home for a minor (presumably, dependent) child, Cox may file as head of household. Because the head-of-household status provides for a larger standard deduction and "wider" tax brackets than does the single status, Cox's most advantageous filing status is as head of household. Choice "1" is incorrect. Cox qualifies as head of household because Cox is not married, is not a surviving spouse, and maintains a home for a minor (presumably, dependent) child. Because the head-of-household status provides for a larger standard deduction and wider tax brackets than does the single status, Cox's most advantageous filing status is as head of household. Choice "2" is incorrect. Cox is not married and therefore cannot file as married filing separately. Choice "3" is incorrect. Cox does not qualify as a surviving spouse because that filing status may only be used in the two years following the death of the spouse, and Cox's spouse died four years ago.

Mort and Mindy met at a New Year's Eve party held December 31, Year 1. They instantly bonded, fell madly in love, and were married at 11:38 p.m that night. Sadly, Mort passed away November 15, Year 2. In January, Year 3, Mindy gave birth to triplets Mark, Mandy, and Maureen. Assuming that Mindy has not remarried, what filing status should she use for Year 4? (1) Single (2) Married filing jointly (3) Head of household (4) Qualifying widow(er) with dependent child

Choice "4" is correct. Because Mindy does not remarry and she maintains a principal residence for her dependent children for the entire year, she may file using the qualifying widow(er) with dependent child status for the two taxable years following Mort's death. In Year 4, the second year after Mort's death, Mindy should file as a qualifying widow.

John created a trust for the benefit of his son, Connor. John does not have the right to change any terms of the trust once established and has no right to income. All trust income is to be distributed to Connor on an annual basis. How should the trust income be reported? (1) To John, reported only on his Form 1040 (2) Reported on Form 1041, with a Schedule K-1 issued to John (3) To Connor, reported only on his Form 1040 (4) Reported on Form 1041, with a Schedule K-1 issued to Connor

Choice "4" is correct. Because the grantor (John) does not retain (1) beneficial enjoyment of the corpus or (2) the power to dispose of the trust income without the approval or consent of any adverse party, the trust is a separate entity for tax purposes. Therefore, the trust would be required to file a Form 1041. The 1041 would issue a schedule K-1 to the income beneficiary, in this case, Connor. Choice "1" is incorrect. Because the grantor did not retain any control or benefit from the trust, the grantor would not be taxed on the trust income. Choice "2" is incorrect. Although the trust would file a Form 1041, a Schedule K-1 would not be issued to John because he is not the income beneficiary. Choice "3" is incorrect. Although the income is reportable by Connor on his individual income tax return, Form 1040 is not the only reporting requirement. The trust will have to file a Form 1041 issuing a Schedule K-1 to Connor.

Thompson' spouse died in Year 1. Thompson did not remarry in year 2 and lived alone the entire year. What is Thompson's Year 2 filing status? (1) Married filing jointly (2) Surviving spouse (3) Head of household (4) Single

Choice "4" is correct. Filing status is determined as of the last day of the year. At the end of Year 2, Thompson is not married and does not qualify for any other filing status. Therefore, his status is single. Choice "1" is incorrect. When a spouse dies during the year, the surviving spouse can file married filing jointly for that year under an exception to the end-of-year test. But this question is about Year 2, the year after death. Thompson is single for Year 2. Choice "2" is incorrect. Surviving spouse status, also known as qualifying widow(er), can be claimed for the two years after year of death. However, it requires the presence of a dependent child, which is not part of the facts here. Thompson is single for Year 2. Choice "3" is incorrect. Head of household status could apply in certain circumstances where there is a dependent and surviving spouse status does not apply. That is not the case here. Thompson is single for Year 2.

Which of the following items must be separately stated on Form 1120S, U.S. Income Tax Return for an S Corporation, Schedule K-1? (1) Mark-to-market income (2) Unearned revenue (3) Section 1245 Gain (4) Gain or loss from the sale of collectibles

Choice "4" is correct. Gain or loss from the S corporation's sale of collectibles is separately reported on the Schedule K-1 of IRS form 1120S. Choice "1" is incorrect. The S corporation's mark-to-market income is part of "ordinary business income (loss)," which is separately stated on the Schedule K-1 of IRS form 1120S. However, the various components of "ordinary business income," such as mark-to-market income, are not separately stated on the K-1 of IRS form 1120S. Choice "2" is incorrect. The S corporation's unearned revenue is not separately stated but is a component of "ordinary business income" or "net rental real estate income (loss)" or "other net rental income (loss)," each of which is separately stated on the Schedule K-1 of IRS form 1120S. However, the various components (such as unearned revenue) of "ordinary business income," "net rental real estate income (loss)," and "other net rental income (loss)" are not separately stated on the K-1 of IRS form 1120S. Choice "3" is incorrect. The S corporation's section 1245 gain (and section 1250 gain) is not separately stated but is a component of "ordinary business income" or "net rental real estate income (loss)" or "other net rental income (loss)," each of which is separately stated on the Schedule K-1 of IRS form 1120S. However, the various components (such as section 1245 gain) of "ordinary business income," "net rental real estate income (loss)," and "other net rental income (loss)" are not separately stated on the K-1 of IRS form 1120S.

Bill and Jane Jones were divorced on January 1, 2018. They have no children. In accordance with the divorce decree, Bill transferred the title of their house over to Jane. The home had a fair market value of $250,000 and was subject to a $100,000 mortgage. Under the divorce agreement, Bill is to make $1,000 monthly mortgage payments on the home for the remainder of the mortgage. In the current year, Bill made 12 mortgage payments. What amount is taxable to Jane in the current year? (1) $12,000 (2) $250,000 (3) $100,000 (4) $0

Choice "4" is correct. If a divorce settlement provides for a property settlement by a spouse, the spouse gets no deduction for payments made and the payments are not includable in gross income of the spouse receiving the payment. Choice "1" is incorrect. Because the divorce settlement provides for the payments, no deduction is allowable for payments made and the payments are not includable in gross income of the spouse receiving the payment. Choice "2" is incorrect. If a divorce settlement provides for a property settlement by a spouse, the amounts are generally nontaxable. Choice "3" is incorrect. If a divorce settlement provides for an assumption of debt by a spouse, the amounts are generally nontaxable.

Pam Petty, age 45, withdrew $10,000 from her traditional IRA. Pam's AGI is $50,000. Pam is employed as a receptionist for the entire year. In which of the following situations would Pam be subject to an early withdrawal penalty? (1) Pam had surgery in the current year and incurred $60,000 in medical costs (2) Pam attended college full-time and paid $12,000 for tuition and books (3) Pam purchased her first home sixty days after withdrawing from her IRA (4) Pam paid $12,000 for medical insurance

Choice "4" is correct. If a taxpayer withdraws money from an IRA before the age of 59½, has received 12 consecutive weeks of unemployment compensation under federal or state law, and purchases medical insurance, there is no penalty for the early withdrawal. Pam is subject to an early withdrawal penalty because she is fully employed. Choice "1" is incorrect. There is no penalty on a premature distribution from an IRA (before age 59½) if the distribution was used to pay for medical expenses in excess of 10% of AGI. Choice "2" is incorrect. There is no penalty on a premature distribution from an IRA (before age 59½) if the distribution was used to pay for college tuition, fees, books, supplies, and equipment and the student attends school at least half-time. Choice "3" is incorrect. There is no penalty on a premature distribution from an IRA (before age 59½) if the distribution was used to pay for a first-time home purchase. The maximum exclusion is $10,000 and the home purchase must be within 120 days of the distribution.

A painted and an accountant agree to trade their services. The painted provides services valued at $550, and the accountant provides services worth $500. What amount should the accountant report as income or expense? (1) $50 expense (2) $50 income (3) $500 income (4) $550 income

Choice "4" is correct. In the case of noncash income, the amount of income to be reported is the fair market value of the property or services received. Since the accountant received services valued at $550, the account must report income of $550. Choice "1" is incorrect. In the case of noncash income, the amount of income to be reported is the fair market value of the property or services received. The fair market value of any services rendered is irrelevant; thus, the difference between the fair market value of services received and the fair market value of services rendered does not result in additional income or expense. Choice "2" is incorrect. In the case of noncash income, the amount of income to be reported is the fair market value of the property or services received. The fair market value of any services rendered is irrelevant; thus, the difference between the fair market value of services received and the fair market value of services rendered does not result in additional income or expense. Choice "3" is incorrect. In the case of noncash income, the amount of income to be reported is the fair market value of the property or services received, not the fair market value of services rendered.

Nolan designed Timber Partnership's new building. Nolan received an interest in the partnership for the services. Nolan's normal billing for these services would be $80,000, and the fair market value of the partnership interest Nolan received is $120,000. What amount of income should Nolan report? (1) $0 (2) $40,000 (3) $80,000 (4) $120,000

Choice "4" is correct. In this question, Nolan receives an interest in the partnership for services performed. The services are valued at the fair market value of what is received (the partnership interest) of $120,000, regardless of what Nolan's normal billing for these services might have been. Choice "1" is incorrect. Nolan would certainly report some income from the services that she performed when something is received in return for those services. Choice "2" is incorrect. The $40,000 is the difference between the fair market value of the partnership (and of the services performed) and Nolan's billing. That number is meaningless in this question. Choice "3" is incorrect. The $80,000 is Nolan's normal billing for her services. However, her income is the $120,000 fair market value of the services. Perhaps, she should charge more. The difference is not considered a gift.

Jackie is 21 years old and is a full-time student at the local community college. She is married to Bill and they have no children. Jackie and Bill live in Jackie's parents' basement while they both finish college. Bill is 25 years old and is also a full-time student. Jackie's parents pay more than half of Jackie and Bill's support. Jackie has no gross income and Bill has $2,000 from a part-time job. Bill and Jackie file a joint return and received a refund because their tax liability is zero. Can Jackie's parents claim Jackie and Bill as dependents on their tax return? (1) Jackie's parents can only claim Jackie, but not Bill as a dependent (2) Jackie's parents can only claim Bill, but not Jackie as a dependent (3) Jackie's parents cannot claim Jackie or Bill as dependents (4) Jackie's parents can claim both Jackie and Bill as dependents

Choice "4" is correct. Jackie qualifies as her parent's qualifying child (CARES). Bill qualifies as Jackie's parents' qualifying relative (SUPORT). Even though Jackie and Bill file a joint return, they can still be considered dependents because they receive a refund and their tax liability is zero.

On February 1 of Year 0, John received a nonqualified stock option to purchase 100 shares of his employer's stock for $10 per share. At the time John received the option, it was selling for $5 per share on an established exchange. On September 1 of Year 1, John exercised the options when the stock was selling for $19 per share. On December 1 of Year 2, John sold all of the shares for $30 per share. What amount and character of income does John recognize in Year 2? (1) $2,000 ordinary income (2) $2,000 long-term capital gain (3) $1,500 ordinary income (4) $1500 long-term capital gain

Choice "4" is correct. John's basis in the stock is $1,500 ($1,000 exercise price + $500 recognized ordinary income). When John received the stock options, he recognized $500 of ordinary income because the stock had an ascertainable market value, which increases his basis in the stock. In Year 2 when John sells the stock, his selling price of $3,000 (100 shares x $30/share) less the adjusted basis of $1,500 equals $1,500. The gain is a long-term capital gain because he held the stock for more than one year. Choices "1" and "2" are incorrect. John must recognize income upon receipt of the stock options because the stock options had an ascertainable market value. The income recognized ($500) increases John's basis in the stock. The gain is a long-term capital gain because he held the stock for more than one year. Choice "3" is incorrect. The gain is a long-term capital gain because John held the stock for more than one year.

On February 1 of Year 0, John received a nonqualified stock option to purchase 100 shares of his employer's stock for $10 per share. At the time John received the option, it was selling for $5 per share on an established exchange. On September 1 of Year 1, John exercised the options when the stock was selling for $19 per share. On December 1 of Year 2, John sold all of the shares for $30 per share. What is the amount and character of income that John must report in Year 2? (1) $2,000 ordinary income (2) $2,000 long-term capital gain (3) $1,500 ordinary income (4) $1,500 long-term capital gain

Choice "4" is correct. John's basis in the stock is $1,500 ($1,000 exercise price + $500 recognized ordinary income). When John received the stock options, he recognized $500 of ordinary income because the stock had an ascertainable market value, which increases his basis in the stock. In Year 2 when John sells the stock, his selling price of $3,000 (100 shares x $30/share) less the adjusted basis of $1,500 equals $1,500. The gain is a long-term capital gain because he held the stock for more than one year. Choices "1" and "2" are incorrect. John must recognize income upon receipt of the stock options because the stock options had an ascertainable market value. The income recognized ($500) increases John's basis in the stock. The gain is a long-term capital gain because he held the stock for more than one year. Choice "3" is incorrect. The gain is a long-term capital gain because John held the stock for more than one year.

Hall, a divorced person and custodian of her 12-year old child, filed her current year federal income tax return as head of a household. She submitted the following information to the CPA who prepared her return: • The divorce agreement, executed in 2017, provides for Hall to receive $3,000 per month, of which $600 is designated as child support. After the child reaches 18, the monthly payments are to be reduced to $2,400 and are to be continued until remarriage or death. However, for the year 2013, Hall received a total of only $5,000 from her former husband. Hall paid an attorney $2,000 in 2012 in a suit to collect the alimony owed. • In June of the current year, Hall's mother gifted her 100 shares of a listed stock. The donor's basis for this stock, which she bought 20 years ago, was $4,000, and market value on the date of the gift was $3,000. Hall sold his stock in July 2013 for $3,500. The donor paid no gift tax. • During the year, Hall spent a total of $1,000 for state lottery tickets. Her lottery winnings in 2013 totaled $200. • Hall earned a salary of $25,000 in the current year. Hall was not covered by any type of retirement plan, but contributed $2,000 to an IRA this year. • During the year, Hall sold an antique that she bought 10 years ago to display in her home. Hall paid $800 for the antique and sold it for $1,400, using the proceeds to pay a court-ordered judgment. • Hall paid the following expenses the current year pertaining to the home that she owns: realty taxes, $3,400; mortgage interest, $7,000; casualty insurance, $490; assessment by city for construction of a sewer system, $910; interest of $1,000 on a personal, unsecured bank loan, the proceeds of which were used for home improvements. Hall does not rent out any portion of the home. What amount should be reported in Hall's current year tax return as alimony income? (1) $36,000 (2) $28,800 (3) $5,000 (4) $0

Choice "4" is correct. None of the payments received should be considered alimony income. Hall would only claim alimony income if total receipts from her former spouse exceeded $7,200 (the required child support). Rule: In the event of payments consisting of both child support and alimony, child support obligations will be satisfied first. Note also that if the divorce was finalized after December 31, 2018, the alimony payments would not be considered income in any situation. Amount designated as monthly child support 600 Number of months × 12 Amount of required child support 7,200 Payments actually received (5,000) Amount of payments considered alimony 0

Michael owns a rental house that generated a $10,000 loss this year. Michael manages the rental property, but does not meet the standards for material participation. Michael is a college professor and has wages of $60,000 and $5,000 in dividend income. How is the $10,000 rental real estate loss treated on Michael's tax return? (1) $5,000 of the loss is deductible against the passive dividend income (2) The rental loss is not deductible because Michael does not have any passive income (3) $10,000 loss is not deductible because Michael does not materially participate in the rental activity (4) $10,000 loss is deductible under the rental real estate exception because Michael actively participates in the rental activity

Choice "4" is correct. Passive activity losses are only deductible against passive activity income. Michael does not have any passive activity income. The dividend income is not considered passive income. An exception for rental real estate allows taxpayers to deduct up to $25,000 of net passive losses attributed to rental real estate in which the taxpayer actively participates. To use this exception, the taxpayer must own at least 10% of the rental activity. Phase-out of the $25,000 allowance begins at $100,000 AGI. Choice "1" is incorrect. An exception for rental real estate allows taxpayers to deduct up to $25,000 of net passive losses attributed to rental real estate in which the taxpayer actively participates, subject to the limitations noted in the answer to choice "4". Choice "2" is incorrect. The exception for rental real estate allows taxpayers to deduct up to $25,000 of net passive losses attributed to rental real estate in which the taxpayer actively participates against nonpassive income. Choice "3" is incorrect. An exception for rental real estate allows taxpayers to deduct up to $25,000 of net passive losses attributed to rental real estate in which the taxpayer actively participates, rather than the more difficult material participation test.

Which statement is correct? (1) A shareholder's basis in an S corporation is not reduced by distributions from the corporation (2) An S corporation may have 120 shareholders (3) A shareholder's basis in an S corporation is increased by the shareholder's guarantee of corporate debt (4) Passive loss limitations may apply to a shareholder's losses from an S corporation

Choice "4" is correct. Passive loss limitations may apply to a shareholder's losses from an S corporation if the S corporation is classified as a passive activity for the shareholder. Choice "1" is incorrect. A shareholder's basis in an S corporation is reduced by distributions from the corporation. Choice "2" is incorrect. An S corporation may not have more than 100 shareholders. Choice "3" is incorrect. A shareholder's basis in an S corporation is not increased by the shareholder's guarantee of corporate debt. Debt basis is only increased by direct loans made by the shareholder to the corporation.

Nare, an accrual-basis taxpayer, owns a building which was rented to Mott under a 10-year lease expiring August 31, Year 8. On January 2, Year 2, Mott paid $30,000 as consideration for canceling the lease. On November 1, Year 2, Nare leased the building to Pine under a 5-year lease. Pine paid Nare $10,000 rent for the two months of November and December, and an additional $5,000 for the last month's rent. What amount of rental income should Nare report in its Year 2 income tax return? (1) $10,000 (2) $15,000 (3) $40,000 (4) $45,000

Choice "4" is correct. Prepaid rent is income when received even for an accrual-basis taxpayer. The $30,000 received as consideration for canceling the lease is in substitution for rental payments and is thus rental income. The $5,000 prepaid for the last month's rent is also rental income. Choice "1" is incorrect. The $30,000 received as consideration for canceling the lease is in substitution for rental payments and is thus rental income. The $5,000 prepaid for the last month's rent is also rental income. Choice "2" is incorrect. The $30,000 is in substitution of rental payments and is thus rental income. Choice "3" is incorrect. The $5,000 prepaid for the last month's rent would also be rental income.

In year 1, Smith, a divorced person, provided over one half the support for his widowed mother, Ruth, and his son, Clay, not of whom as US citizens. During year 1, Ruth did not live with Smith. She received $9000 in social security benefits. Clay, a 25 year old full time graduate student, and his wife lived with Smith. Clay had no income but filed a joint return for year 1, owing an additional $500 on taxes on his wife's income. How many people meet the definition of either qualifying child or qualifying relative for Smith? (1) 0 (2) 3 (3) 2 (1) 1

Choice "4" is correct. Ruth is the only one who meets the definition of a qualifying relative for Smith. Ruth's $9,000 Social Security income is considered to be under the gross income limitation for QR because it is nontaxable income and is not considered for the gross income limitation. Clay is over the age limit for qualifying child. Clay files a joint return (with a tax liability) with his wife, so he also does not meet the definition of qualifying relative. Choice "1" is incorrect. Neither Clay nor his wife meet the definition of qualifying child or qualifying relative. Clay is over the age limit for qualifying child purposes. The fact that Clay and his wife file a joint return (with a tax liability) prevents them from meeting the definition of qualifying relatives. Choice "2" is incorrect. Neither Clay nor his wife meet the definition of qualifying child or qualifying relative. Clay is over the age limit for qualifying child purposes. The fact that Clay and his wife file a joint return (with a tax liability) prevents them from meeting the definition of qualifying relatives. Choice "3" is incorrect. Neither Clay nor his wife meet the definition of qualifying child or qualifying relative. Clay is over the age limit for qualifying child purposes. The fact that Clay and his wife file a joint return (with a tax liability) prevents them from meeting the definition of qualifying relatives.

Bill and Anne Chambers are married and file a joint return. They have no children. Their college friend Ryan lived with them for the entire current tax year. Ryan is 40 years old and earned $2,000 at a part-time job and received $25,000 in municipal bond interest. Ryan is a citizen of the United States and is unmarried. Which of the following statements is true regarding claiming Ryan as a dependent on the Chamberses' tax return? (1) If Ryan earned $15,000 in self-employment income in addition to the part-time job and municipal bond interest, he would qualify as a dependent on Chambers' tax return (2) Ryan qualifies as a dependent for the Chambers under the qualifying child rules (3) Ryan qualifies as a dependent for the Chambers under the qualifying relative rules because he lived with the Chambers for the entire year, as long as Ryan does not provide more than half of his own support (4) Ryan qualifies as a dependent for the Chambers under the qualifying relative rules as long as the Chambers provide more than half of Ryan's support

Choice "4" is correct. Ryan meets the SUPORT tests for a qualifying relative if the Chambers provide more than half of Ryan's support. CARES Test (Qualifying Child) Close Relative Age Limit Residency and Filing Requirements Eliminate Gross Income Test Only citizens (residents of U.S./Canada or Mexico) test Support Test SUPORT Test (Qualifying Relative) Support (over 50%) testRelative test Under a specific amount of (taxable) gross income test Precludes dependent filing a joint tax return test Only citizens (residents of U.S./Canada or Mexico) test Relative test Ryan's taxable income ($2,000) is under the gross income threshold amount. He doesn't file a joint return. He is a citizen of the United States and he lives with the Chambers for the entire year. Choice "1" is incorrect. If Ryan earns $15,000 in self-employment income, he would not meet the "U" test because his taxable income is over the gross income threshold amount. Choice "2" is incorrect. Ryan does not meet the CARES test for qualifying child. He is not a close relative and not under the age limit. Choice "3" is incorrect. The support test for the qualifying relative test states that the Chambers must pay more than half of the support of Ryan. The fact that Ryan does not provide more than half of his own support is not enough to meet the support test for qualifying relative (SUPORT).

Ryan is 39 years old and works as a real estate agent. Ryan has a traditional (deductible) IRA with a current balance of $80,000. The IRA consists of $60,000 of contributions that Ryan made and deducted on his tax return and $20,000 of account earnings. In the current year, Ryan receives a distribution of the entire $80,000. He contributes $60,000 to a Roth IRA and keeps the remaining $20,000. What is Ryan's total income tax penalty on the transactions? (1) $0 income tax, $0 penalty (2) $20,000 income tax, $8,000 penalty (3) $5,000 income tax, $2,000 penalty (4) $20,000 income tax, $2,000 penalty

Choice "4" is correct. Ryan will pay income tax on the entire withdrawal ($80,000 x 25% = $20,000). He will pay a penalty on the portion of the distribution that he did not contribute to the Roth IRA ($20,000 x 10% = $2,000).

Jason is a cash basis, self-employed attorney. Which of the following expenses are deductible on his Schedule C? (1) Mrs. Jones was billed in February of the prior year, but has never paid. Jackson considers this to be bad debt (2) Jason made a $1,000 contribution to the Boys and Girls Club (3) Jason paid country club dues totaling $4,000 for the year. Jason meets potential and current clients at the country club (4) Jason paid$35,000 to his part-time assistant

Choice "4" is correct. Salaries and wages paid to employees are deductible business expenses on Schedule C of Form 1040. Choice "1" is incorrect. Because Jason is a cash basis taxpayer, he has not reported the income from Mrs. Jones. Therefore, the fact that she hasn't paid him does not create a deductible expense for Jason. Choice "2" is incorrect. Charitable contributions should be reported on Schedule A as an itemized deduction. Choice "3" is incorrect. Country club dues are not deductible business expenses.

With regard to S Corporations and their stockholders, the "at-risk" rules applicable to losses: (1) Depend on the type of income reported by the S corporation (2) Are subject to the elections made by the S corporation's stockholders (3) Take into consideration the S corporation's ratio of debt to equity (4) Apply at the shareholder level rather than at the corporate level

Choice "4" is correct. The "at risk" rules apply at the shareholder level rather than at the corporate level. Rule: The "at risk" rules limit the deductibility of the distributive share of the losses of an S corporation to the amount the taxpayer has "at risk" (as opposed to non recourse loans) and could actually lose from an activity. Choice "1" is incorrect. The character (type) of an S corporation's income only affects the (type) character of flow through to stockholders. Choice "2" is incorrect. A far out distractor. The elections that affect the nature and timing of an S corporation income are made at the corporate level, not the shareholder level. The "at risk" provisions afford no elections. Choice "3" is incorrect. A far out distractor. The S corporation's ratio of debt to equity is only an indication of whether or not the corporation has been properly capitalized.

Larry received the following dividends in the current tax year: $1,000 cash from Bears, Inc. $500 (FMV) of property with a $200 basis from Tigers Inc. Stock dividend from Lions Inc. of 50 shares ($15/share FMV) All three corporation are publicly traded. Larry had the option to receive cash instead of a stock dividend from Lions Inc. What amount of the dividends received must Larry include in gross income on his federal tax return for the current year? (1) $1,000 (2) $1,200 (3) $1,500 (4) $2,250

Choice "4" is correct. The $1,000 cash dividend from Bears Inc. is taxable. The $500 FMV of the property received as a dividend from Tigers Inc. is taxable. Because Larry had the option to receive cash instead of the stock dividend from Lions Inc., the $750 FMV of the stock dividend is taxable. Consequently, the total taxable dividends equal $1,000 + $500 + $750 = $2250. Choice "1" is incorrect. The $500 FMV of the property received as a dividend from Tigers Inc. is taxable generally taxable as are cash dividends in lieu of stock. Choice "2" is incorrect. The property distribution is valued at the FMV of the property received as a dividend, rather than the adjusted basis. Also, because Larry had the option to receive cash instead of the stock dividend from Lions Inc., the $750 FMV of the stock dividend is taxable. Choice "3" is incorrect. Because Larry had the option to receive cash instead of the stock dividend from Lions Inc., the $750 FMV of the stock dividend is taxable.

Nia Johnson invested in a certificate of deposit (CD) at the local bank. The total interest to be earned on the CD amounted to $1,000. However, Nia withdrew the money early and only earned $800. The bank reported $1,000 of interest and a $200 early withdrawal penalty to Nia for tax reporting. How will Nia report the interest earned and the early withdrawal penalty? (1) $800 as interest income (2) $1,000 as interest income and no deduction for the early withdrawal penalty (3) $1,000 as interest income and a $200 itemized deduction for the early withdrawal penalty (4) $1,000 as interest income and a $200 adjustment to AGI for the early withdrawal penalty

Choice "4" is correct. The $1,000 interest income is reported in gross income and a $200 adjustment is taken for the forfeited interest due to withdrawing the money early from the investment. Choice "1" is incorrect. The interest income and penalty should not be netted. Instead, the interest income of $1,000 is reported as interest, with a separate $200 adjustment for the early withdrawal penalty. Choice "2" is incorrect. The $1,000 interest income is reported in gross income. The $200 penalty for withdrawing the money early is an adjustment to AGI rather than an itemized deduction. Choice "3" is incorrect. The $1,000 interest income is reported in gross income and the $200 early withdrawal penalty is deducted as an adjustment, not an itemized deduction.

A taxpayer's spouse dies in August of the current year. Which of the following is the taxpayer's filing status for the current year? (1) Single (2) Qualified widow(er) (3) Head of household (4) Married filing jointly

Choice "4" is correct. The surviving spouse is considered to be married (and thus able to file as married filing jointly) for the entire current year even if the spouse dies earlier in the year (in this case in August). Choice "1" is incorrect. The filing status is not single for the current year. Choice "2" is incorrect. The filing status is not qualified widow(er) for the current year. Choice "3" is incorrect. The filing status is not head of household for the current year.

The Uniform Capitalization Rules of Code Sec. 263A apply to retailers those average gross receipts for the preceding three years exceed what amount? (1) $1,000,000 (2) $5,000,000 (3) $10,000,000 (4) $26,000,000

Choice "4" is correct. The uniform capitalization rules do not apply to inventory acquired for resale if the taxpayer's average gross receipts for the preceding three tax years do not exceed $26,000,000.

Robert Corp. granted an incentive stock option for 200 shares to Beverly, an employee, on March 14, Year 12. The option price and FMV on the date of grant was $150. Beverly exercised the option on August 2, Year 14, when the FMV was $180 per share. She sold the stock on September 20, Year 15, for $250 per share. How much gross income did Beverly recognize in Year 15? (1) $30,000 (2) $150 (3) $0 (4) $20,000

Choice "4" is correct. This is the gain Beverly will recognize upon the sale of the stock. The purchase was 200 shares at $150 per share, or $30,000. The sale was 200 shares at $250 per share, or $50,000. This gain on incentive stock options is not recognized until the sale occurs in Year 15. Choice "1" is incorrect. This is simply the purchase price of the stock upon exercise of 200 shares at $150 per share. Choice "2" is incorrect. This is simply the option price per share on the date of grant. Choice "3" is incorrect. The realized gain on the sale must be recognized in the year of the sale per the above explanation.

Easel Co. has elected to reimburse employees for business expenses under a nonaccountable plan. Easel does not require employees to provide proof of expenses and allows employees to keep any amount not spent. Under the plan, Mel, an Easel employee for a full year, gets $400 per month for business automobile expenses. At the end of the year Mel informs Easel that the only business expense incurred was for business mileage of 12,000 at a rate of 30 cents per mile, the IRS standard mileage rate at the time. Mel encloses a check for $1,200 to refund the overpayment to Easel. What amount should be reported in Mel's gross income for the year? (1) $0 (2) $1,200 (3) $3,600 (4) $4,800

Choice "4" is correct. Under a nonaccountable plan, $4,800 ($400 per month x 12 months) must be reported as part of Mel's gross income for the year (in fact, the $4,800 will be included as part of Mel's taxable wages on Mel's W-2). Rule: Under a nonaccountable plan (i.e., expenses are not reported to the employer), any amounts received by an employee from the employer must be reported by the employer as part of wages on the employee's W-2 for the year (and subject to income tax withholding requirements). The gross amount received is reported as income.

Under the uniform capitalization rules applicable to property acquired for resale, which of the following costs should be capitalized with respect to inventory if no exceptions are met? Marketing costs; Off-site storage costs (1) Yes; Yes (2) Yes; No (3) No; No (4) No; Yes

Choice "4" is correct. Under the uniform capitalization rules, purchasers of inventory for resale may deduct their marketing costs but must capitalize their off-site storage costs. Choices "1", "2", and "3" are incorrect. Marketing costs are deductible, but off-site storage must be capitalized.

Which of the following is subject to the Uniform Capitalization Rules of Code Sec. 263A? (1) Editorial costs incurred by a freelance writer (2) Research and experimental expenditures (3) Mine development and exploration costs (4) Warehousing costs incurred by a manufacturing company with $12 million in annual gross receipts

Choice "4" is correct. Uniform capitalization rules apply to the following: (1) real or tangible personal property produced by the taxpayer for use in his or her trade or business; (2) real or tangible personal property produced by the taxpayer for sale to his or her customers; and (3) real or tangible personal property acquired by the taxpayer for resale, provided the taxpayer's annual average gross receipts for the preceding three years exceeds $26,000,000. Warehousing costs incurred by a manufacturing company (making inventory for sale to its customers) are subject to the Uniform Capitalization Rules. Further, they are the only item on the list that is real or tangible personal property. In this case, the inventory is not acquired for resale (it is produced by the taxpayer for sale to his or her customers), so the fact that the annual sales are $32,000,000 does not matter in this case. The sales could have been less than $26,000,000 annually, and the Uniform Capitalization Rules would still have applied.

Which one of the following will result in an accruable expense for an accrual-basis taxpayer? (1) An invoice dated prior to year end but the repair completed after year end (2) A repair completed prior to year end but not invoiced (3) A repair completed prior to year end and paid upon completion (4) A signed contract for repair work to be done and the work is to be completed at a later date

RULE: An accruable expense is one is which the services have been received/performed but have not been paid for by the end of the reporting period. Choice "2" is correct. The facts indicate that a repair was completed prior to year end but not yet invoiced. If it has not yet been invoiced, it is assumed that it has also not yet been paid for. Therefore, this is a situation in which the repair expense would be accrued at year end. Services have been performed, but they have not been paid for, as they have not even been invoiced yet. Choice "1" is incorrect. If the repair was completed after year end, then the expense is not accruable, as the benefit of the services hasn't been received as of year end. The fact that the repair was invoiced prior to year end does not impact the situation. Choice "3" is incorrect. If a repair was completed and paid for prior to year end, no accrual is appropriate. On the accrual basis, the expense is taken in the year the repair is completed and the benefit is received. In this case, the account payable was also paid in the same year, but this has no effect on the expense. Choice "4" is incorrect. The facts indicate that the work is to be completed at a date later than year end. Therefore, the expense is not accruable at year end, as the benefit of the repair hasn't been received as of year end. It is reasonable that a signed contract for the repair work exists, but this has no effect on the accrual.

Porter was unemployed for part of the year. Porter received $35,000 of wages, $6,400 from a state unemployment compensation plan, and $2,000 from his former employer's company-paid supplemental unemployment benefit plan. What is the amount of Porter's gross income? (1) $35,000 (2) $37,000 (3) $41,400 (4) $43,400

RULE: Gross income includes all income unless it is specifically excluded in the tax code. Choice "4" is correct. Wages and all unemployment compensation are not excluded from being taxable; therefore, they are included in the taxpayer's gross income for tax purposes. Wages received 35,000 State unemployment compensation 6,400 Employer's unemployment compensation plan 2,000 43,400 Choice "1" is incorrect. All forms of unemployment compensation are included as part of gross income. Choice "2" is incorrect. The $6,400 of state unemployment compensation received is included as part of gross income. Choice "3" is incorrect. The $2,000 of his former employer's company-paid supplemental unemployment benefit plan is included as part of gross income.

Barkley owns a vacation cabin that was rented to unrelated parties for 10 days during the year for $2,500. The cabin was used by Barkley for three months and left vacant for the rest of the year. Expenses for the cabin were as follows: Real estate taxes $1,000 Maintenance and utilities $2,000 How much rental income (loss) is included in Barkley's adjusted gross income? (1) $0 (2) $500 (3) $(500) (4) $(1,500)

RULE: If a vacation residence is rented for less than 15 days per year, it is treated as a personal residence. The rental income is excluded from income, and mortgage interest (first or second home) and real estate taxes are allowed as itemized deductions. Depreciation, utilities, and repairs are not deductible. Choice "1" is correct. Applying the RULE above, if a vacation residence is rented for less than 15 days per year, it is treated as a personal residence. The rental income ($2,500 in this case) is excluded from income. A Schedule E is not filed for this property (i.e., no income is reported, the taxes are reported as itemized deductions, and the maintenance and utilities are not deductible), so the effect on AGI is zero. Choice "2" is incorrect. This assumes that the property taxes are reported as itemized deductions but that the rental income ($2,500) less the maintenance and utilities ($2,000) are reported net on Schedule E. Per the above RULE, the rental income is excluded from income, and the maintenance and utilities are not deductible. Choice "3" is incorrect. This assumes that all of the items shown are reported net on the Schedule E-$2,500 - $1,000 - $2,000 = ($500). Per the above RULE, the rental income is excluded from income, the maintenance and utilities are not deductible, and the property taxes are reported on Schedule A as an itemized deduction. Choice "4" is incorrect, per the above RULE and discussion.

In which of the following situations may taxpayers file as married filing jointly? (1) Taxpayers who were married but lived apart during the year (2) Taxpayers who were married but lived under a legal separation agreement at the end of the year (3) Taxpayers who were divorced during the year (4) Taxpayers who were legally separated but lived together for the entire year

RULE: In order to file a joint return, the parties must be MARRIED at the end of the year. Exception: If the parties are married but are LEGALLY SEPARATED under the laws of the state in which they reside, they cannot file a joint return (they will file either under the single or head of household filing status). Choice "1" is correct. Per the above rule, taxpayers who are married but lived apart during the year are allowed to file a joint return for the year. The fact that they did not live together during the year has no bearing on the issue. Choice "2" is incorrect. Per the above rule, taxpayers who are married but lived under a legal separation agreement at the end of the year may not file a joint return. They will generally file either under the single or head of household filing status. Choice "3" is incorrect. Per the above rule, taxpayers who were divorced during the year may not file a joint return together, as they are not married at the end of the year. [Note, however, that they may become married again in the year and file a joint return with the new spouse.] Choice "4" is incorrect. Per the above rule, taxpayers who were legally separated but lived together for the entire year may not file a joint return. They will generally file either under the single or head of household filing status.

Lane, a single taxpayer, received $160,000 in salary, $15,000 in income from an S corporation in which Lane does not materially participate, and a $35,000 passive loss from a real estate rental activity in which Lane materially participated. Lane's modified adjusted gross income was $165,000. What amount of the real estate rental activity loss was deductible? (1) $0 (2) $15,000 (3) $25,000 (4) $35,000

Rule: Passive activity is any activity in which the taxpayer does not materially participate. A net passive activity loss generally may not be deducted against other types of income (e.g., wages, other ordinary or active income, portfolio income (interest and dividends), or capital gains). In other words, passive losses may generally only offset passive income for a tax year-the remaining net loss is generally "suspended" and carried forward to a year when it may be used to offset passive income (or when the final disposition of the property occurs). However, there is an exception (the "mom and pop exception," as we refer to it in the textbooks) to this general rule. Taxpayers who own more than 10% of the rental activity, have modified AGI under $100,000, and have active participation (managing the property qualifies), may deduct up to $25,000 annually of net passive losses attributable to real estate. There is a phase-out provision for modified AGI from $100,000 − $150,000, and the deduction is completely phased-out for modified AGI in excess of $150,000. Choice "2" is correct. Per the above rule, unless an exception exists (and it does not in this case, as Lane's modified adjusted gross income is in excess of $150,000), passive losses may only offset passive income for a tax year (i.e., no "net loss" may exist). In this case, Lane has a $20,000 net loss from passive activity [$15,000 S Corporation income (passive, in this case because the facts state Lane does not materially participate) minus the $35,000 rental real estate loss]. Thus, only $15,000 of the passive loss from real estate rental activity may be used to offset the $15,000 income from the S Corporation. The remaining $20,000 passive activity loss is carried forward to be used in future years. Choice "1" is incorrect. Per the above rule, passive losses may generally only offset passive income for a tax year. Lane has passive income of $15,000 in the year; thus, passive loss up to $15,000 may be deducted from passive income. Choice "3" is incorrect. This answer option is an attempt to confuse the candidate into using the "mom and pop" exception, which applies when taxpayers who actively participate, own more than 10% of the rental activity, and have modified AGI under $100,000 are able to deduct up to $25,000 annually of net passive losses attributable to real estate. There is a phase-out provision for modified AGI from $100,000 − $150,000, and the deduction is completely phased-out for modified AGI in excess of $150,000. In this case, the facts state that Lane's modified adjusted gross income is $165,000; thus, Lane does not qualify to use the exception. Choice "4" is incorrect. This answer option assumes that the full amount of the rental real estate loss is deductible against the passive income from the S Corporation, and, thus, against Lane's other taxable income. As indicated in the rule above, unless an exception applies (it does not in this case), a net passive activity loss may not be deducted against other types of income (e.g., wages, other ordinary or active income, portfolio income (interest and dividends), or capital gains). Thus, the full $35,000 rental real estate loss is not deductible in the year by Lane.

Kant, a cash-basis individual, owns and operates an office building. Kant received the following payments during the current year: Current rents $30,000 Advance rents for the next year 10,000 Security deposits held in a segregated account 5,000 Lease cancellation payments 15,000 What amount is included in gross income? (1) $30,000 (2) $40,000 (3) $55,000 (4) $60,000

Rule: The basic formula for determination of net rental income or loss follows: · Gross rental income · Prepaid rental income · Rent cancellation payments · Improvements in lieu of rent · (Rental expenses) · Net rental income (loss) If security deposits are held separately and not available to be applied to last month's rent (as in a segregated account), they are a liability of the taxpayer and not included in income in the year received. Choice "3" is correct. The calculation of gross income for the year follows: Current rents 30,000 Advance rents for the next year 10,000 Security deposits held in a segregated account − Lease cancellation payments 15,000 Gross income from the rental activity 55,000 Choice "1" is incorrect. This answer option incorrectly includes only the current rents as part of gross income, when advance rents and lease cancellation payments also must be included. Choice "2" is incorrect. This answer option incorrectly includes only the current rents and the advance rents as part of gross income, when lease cancellation payments also must be included. Choice "4" is incorrect. This answer option incorrectly includes all of the payments collected for the rental activity in the year, when the security deposits that are held in a segregated account are excluded from gross income.

An individual received $50,000 during the current year pursuant to a divorce decree executed in 2015. A check for $25,000 was identified as annual alimony, checks totaling $10,000 as annual child support, and a check for $15,000 as a property settlement. What amount should be included in the individual's gross income? (1) $50,000 (2) $40,000 (3) $25,000 (4) $0

Rules: Payments for the support of a spouse are income to the spouse receiving the payments and are deductible to arrive at adjusted gross income by the contributing spouse. Child support is not taxable. Property settlements are not taxable. Choice "3" is correct. Only the $25,000 in alimony is included in the gross income of the receiving spouse. Choice "1" is incorrect. This answer option incorrectly includes all of the payments received in the year. The child support ($10,000) and the property settlement ($15,000) are NOT included in the gross income of the receiving spouse. Choice "2" is incorrect. This answer option incorrectly includes the payments received in the year for alimony and property settlement for the year [$25,000 + $15,000 = $40,000]. The property settlement ($15,000) is NOT included in the gross income of the receiving spouse. Choice "4" is incorrect. The amount received for alimony ($25,000) is included in the gross income of the receiving spouse.

Merrill and Joe's divorce was finalized in June of 2012. As part of the settlement, Joe received the following: Alimony $3,000/per month Child support 1,000/per month Lump-sum payment as the property settlement 125,000 Payments began in July; however, Merrill only paid a total of $15,000 during the year. For the current year, what amount must Joe include in income on his Form 1040? (1)$9,000 (2) $15,000 (3) $134,000 (4) $140,000

hoice "1" is correct. Alimony (received pursuant to a divorce agreement executed on or before December 31, 2018) is an item of gross income; child support is not. Alimony paid according to a divorce agreement executed after December 31, 2018, is neither taxable to the recipient nor deductible by the payor (Tax Cuts and Jobs Act of 2017). Because this divorce was finalized in 2012, the alimony is included in gross income. Joe was to receive $3,000 per month in alimony for the remaining six months of the year (July - December), for a total of $18,000. Child support is non-taxable as are lump-sum property settlements made pursuant to a divorce. When total payments received do not equal the total due, the amounts are first allocated to child support. Thus, of the $15,000 paid by Merrill, $6,000 is first allocated to child support. The remaining $9,000 would constitute alimony and would be taxable. Choice "2" is incorrect. The $15,000 must first be allocated between the types of payments received. Any amounts are first used to satisfy any child support requirement, and the remainder would be classified as alimony. Choice "3" is incorrect. This answer includes both the $9,000 (discussed above) and the property settlement (which is non-taxable). Choice "4" is incorrect. This answer includes the total amount received, $15,000 payments (child support and alimony) and the property settlement. Lump-sum property settlements are not taxable to the recipient in a divorce.


Ensembles d'études connexes

Riboswitches and Gene Regulation in Development (Lect. 19)

View Set

Types of life insurance policies

View Set

Network+ 8th edition, Chapter 1 review questions, Chapter 5: Cabling, Ch 4: Chapter Review Questions, Chapter 3: Addressing on Networks, Computer Network CH2 Review 8th Edition

View Set

Computer Network Fundamentals TestOut Pro chapter 11

View Set

Biology Chapter 53 Population Ecology

View Set

Waves Study Guide warm ups and vocab

View Set

Introduction to IT - C182 - Pre-Assessment

View Set

Nester 7e Microbiology Chapter 1 Humans and the Microbial World

View Set